TASC Reading Practice Test

Try our free TASC Reading practice test. These practice questions are designed to help you prepare for the reading section of the TASC high school equivalency test. For this section you will be presented with passages of text that are followed by questions about the key ideas, the craft and structure of the passage, and word usage. There are 50 questions which must be answered within 75 minutes.

Congratulations - you have completed .

You scored %%SCORE%% out of %%TOTAL%%.

Your performance has been rated as %%RATING%%


Your answers are highlighted below.
Question 1

Wu Mei

Our view of early history blends written records with legendary stories. Over the ages, documents are lost, tales are told, and new legends emerge. The story of Wu Mei is one such tale. Wu Mei grew up in China in the 1600s, training hard in martial arts in the Shaolin temple. Because of her dedication and skill, Mei eventually became one of the Five Elders of the temple, a guiding figure for others who trained there. Sadly, the temple was destroyed in a war during the Qing Dynasty. Mei escaped, however, and went on to develop a new form of martial arts that could be used by common people after only a short amount of training.

According to legend, Mei developed the new martial arts form after meeting a beautiful young girl who was being forced to marry a bandit. Mei trained the young girl to defend herself so she could not be forced to marry him. Because they had little time to train and because they needed to do it in secret, Mei created a style of martial arts that was useful to smaller or weaker fighters. This form of fighting became known as Wing Chun.

To further improve the fighting style, Mei drew inspiration from the movements of the animals she encountered in daily life. She especially favored the crane's graceful and stealthy motions. By basing her fighting style on animals that were common in all of China, Mei made her fighting technique easy to learn for the masses. Mei's fighting style remains in use even today. Mei's powerful martial arts skills and her understanding of others' struggles let her have a lasting impact in the martial arts world.
 

Which of the following would make the best title for this selection?

A
Martial Arts in China during the Qing Dynasty
B
A Brief History of Shaolin Temples
C
Wu Mei: Bringing Martial Arts to the Masses
D
Wu Mei: How Legends Become Reality
Question 1 Explanation: 
The correct answer is (C). The passage summarizes events in Wu Mei’s life that led her to create a style of martial arts that was accessible to the masses. The passage does not discuss other forms of martial arts (A), or the difference between the legend of Wu Mei and who she really was (D). Answer (B) is incorrect because Shaolin Temples are mentioned in the passage, but not explored in detail.
Question 2

Wu Mei

Our view of early history blends written records with legendary stories. Over the ages, documents are lost, tales are told, and new legends emerge. The story of Wu Mei is one such tale. Wu Mei grew up in China in the 1600s, training hard in martial arts in the Shaolin temple. Because of her dedication and skill, Mei eventually became one of the Five Elders of the temple, a guiding figure for others who trained there. Sadly, the temple was destroyed in a war during the Qing Dynasty. Mei escaped, however, and went on to develop a new form of martial arts that could be used by common people after only a short amount of training.

According to legend, Mei developed the new martial arts form after meeting a beautiful young girl who was being forced to marry a bandit. Mei trained the young girl to defend herself so she could not be forced to marry him. Because they had little time to train and because they needed to do it in secret, Mei created a style of martial arts that was useful to smaller or weaker fighters. This form of fighting became known as Wing Chun.

To further improve the fighting style, Mei drew inspiration from the movements of the animals she encountered in daily life. She especially favored the crane's graceful and stealthy motions. By basing her fighting style on animals that were common in all of China, Mei made her fighting technique easy to learn for the masses. Mei's fighting style remains in use even today. Mei's powerful martial arts skills and her understanding of others' struggles let her have a lasting impact in the martial arts world.
 

Which event came last?

A
The Shaolin Temple was destroyed.
B
Mei created a style of martial arts for smaller, weaker fighters.
C
Mei became one of the temple’s Five Elders.
D
Mei trained a young girl to defend herself.
Question 2 Explanation: 
The correct answer is (B). The first paragraph of the passage lays out a pretty specific chronology. Mei was already one of the Shaolin Temple’s Five Elders when the temple was destroyed. Her experience training a young girl to defend herself is what led to the creation of the Wing Chung style.
Question 3

Wu Mei

Our view of early history blends written records with legendary stories. Over the ages, documents are lost, tales are told, and new legends emerge. The story of Wu Mei is one such tale. Wu Mei grew up in China in the 1600s, training hard in martial arts in the Shaolin temple. Because of her dedication and skill, Mei eventually became one of the Five Elders of the temple, a guiding figure for others who trained there. Sadly, the temple was destroyed in a war during the Qing Dynasty. Mei escaped, however, and went on to develop a new form of martial arts that could be used by common people after only a short amount of training.

According to legend, Mei developed the new martial arts form after meeting a beautiful young girl who was being forced to marry a bandit. Mei trained the young girl to defend herself so she could not be forced to marry him. Because they had little time to train and because they needed to do it in secret, Mei created a style of martial arts that was useful to smaller or weaker fighters. This form of fighting became known as Wing Chun.

To further improve the fighting style, Mei drew inspiration from the movements of the animals she encountered in daily life. She especially favored the crane's graceful and stealthy motions. By basing her fighting style on animals that were common in all of China, Mei made her fighting technique easy to learn for the masses. Mei's fighting style remains in use even today. Mei's powerful martial arts skills and her understanding of others' struggles let her have a lasting impact in the martial arts world.
 

As it is used in the first paragraph, the word “Elders” most nearly means

A
old people living in the Shaolin Temple.
B
experts in Wu Mei’s fighting style.
C
the leaders of the Shaolin Temple.
D
leaders in China during the Qing Dynasty.
Question 3 Explanation: 
The correct answer is (C). The passage explains that as one of the five Elders, Wu Mei was a “guiding figure” for others who trained there, making her something of a leader. There is no indication from the passage that the “Five Elders” are elderly (A), nor is there any indication that the title refers to anyone outside of the Shaolin Temple (D). Since Wu Mei developed her fighting style after the temple was destroyed, it’s illogical for anyone at the temple to be an expert in her fighting style (B).
Question 4

Wu Mei

Our view of early history blends written records with legendary stories. Over the ages, documents are lost, tales are told, and new legends emerge. The story of Wu Mei is one such tale. Wu Mei grew up in China in the 1600s, training hard in martial arts in the Shaolin temple. Because of her dedication and skill, Mei eventually became one of the Five Elders of the temple, a guiding figure for others who trained there. Sadly, the temple was destroyed in a war during the Qing Dynasty. Mei escaped, however, and went on to develop a new form of martial arts that could be used by common people after only a short amount of training.

According to legend, Mei developed the new martial arts form after meeting a beautiful young girl who was being forced to marry a bandit. Mei trained the young girl to defend herself so she could not be forced to marry him. Because they had little time to train and because they needed to do it in secret, Mei created a style of martial arts that was useful to smaller or weaker fighters. This form of fighting became known as Wing Chun.

To further improve the fighting style, Mei drew inspiration from the movements of the animals she encountered in daily life. She especially favored the crane's graceful and stealthy motions. By basing her fighting style on animals that were common in all of China, Mei made her fighting technique easy to learn for the masses. Mei's fighting style remains in use even today. Mei's powerful martial arts skills and her understanding of others' struggles let her have a lasting impact in the martial arts world.
 

According to the passage, in what way did Wu Mei’s fighting style benefit from the fact that she based it on the movements of animals around China?

A
It helped her students feel connected with nature.
B
It capitalized on what had enabled animals to survive among humans.
C
It highlighted the Qing Dynasty’s great reverence for animals.
D
It helped her fighting style appeal to people all over China.
Question 4 Explanation: 
The correct answer is (D). The selection states, “By basing her fighting style on animals that were common in all of China, Mei made her fighting technique easy to learn for the masses.” In other words, her style was easy to learn because the movements were based on animals that most people were likely to recognize.
Question 5

Wu Mei

Our view of early history blends written records with legendary stories. Over the ages, documents are lost, tales are told, and new legends emerge. The story of Wu Mei is one such tale. Wu Mei grew up in China in the 1600s, training hard in martial arts in the Shaolin temple. Because of her dedication and skill, Mei eventually became one of the Five Elders of the temple, a guiding figure for others who trained there. Sadly, the temple was destroyed in a war during the Qing Dynasty. Mei escaped, however, and went on to develop a new form of martial arts that could be used by common people after only a short amount of training.

According to legend, Mei developed the new martial arts form after meeting a beautiful young girl who was being forced to marry a bandit. Mei trained the young girl to defend herself so she could not be forced to marry him. Because they had little time to train and because they needed to do it in secret, Mei created a style of martial arts that was useful to smaller or weaker fighters. This form of fighting became known as Wing Chun.

To further improve the fighting style, Mei drew inspiration from the movements of the animals she encountered in daily life. She especially favored the crane's graceful and stealthy motions. By basing her fighting style on animals that were common in all of China, Mei made her fighting technique easy to learn for the masses. Mei's fighting style remains in use even today. Mei's powerful martial arts skills and her understanding of others' struggles let her have a lasting impact in the martial arts world.
 

Which anecdote best supports the main idea introduced in the opening sentences?

A
The legend of Mei training a young girl
B
The fact that Mei based her style of martial arts on animals
C
The story of the Shaolin Temple being destroyed
D
The detail about Mei becoming an Elder of the Shaolin temple
Question 5 Explanation: 
The correct answer is (A). This question requires you to find supporting evidence for the idea presented in the opening sentences, which discuss how legends and stories can blend into our understanding of history. The story about the young girl begins with the phrase, “According to legend,” which supports the idea that legends and stories influence our view of historical figures.
Question 6

Differing Views
by Annalise Gulstad

Bubbles squeak out from beneath my feet,
My toes mingle with the mud,
The tiny creek babbles beside me.

Above my head a little brown wren chatters angrily down,
Her chirps of frustration do little along the lines of moving me along,
I breathe out and see her chirping as part of nature’s song.

Pondering her view of the forest creek as compared to mine I consider, What a great beast I must seem, tromping through the mud,
Unlike the deer each of my footfalls lands with a thud.

Meanwhile she flits around between the branches,
Silent but for her song which she raises in fear,
Snatching bugs between the rough bark.

What I had found to be a sweet reprieve,
She perceived as a dangerous siege.

 

The phrase “tiny creek babbles” is an example of

A
personification
B
hyperbole
C
a simile
D
irony
Question 6 Explanation: 
The correct answer is (A). Personification is attributing human characteristics to something that is nonhuman. A brook can flow and move, and a brook can make noise, but “babbling” is something only a talkative human can do.
Question 7

Differing Views
by Annalise Gulstad

Bubbles squeak out from beneath my feet,
My toes mingle with the mud,
The tiny creek babbles beside me.

Above my head a little brown wren chatters angrily down,
Her chirps of frustration do little along the lines of moving me along,
I breathe out and see her chirping as part of nature’s song.

Pondering her view of the forest creek as compared to mine I consider, What a great beast I must seem, tromping through the mud,
Unlike the deer each of my footfalls lands with a thud.

Meanwhile she flits around between the branches,
Silent but for her song which she raises in fear,
Snatching bugs between the rough bark.

What I had found to be a sweet reprieve,
She perceived as a dangerous siege.

 

What is implied about the speaker from her experience?

A
She does not feel comfortable in her own skin.
B
She chatters as much as the wren does.
C
She knows the animals have a unique perspective.
D
She is damaging the forest by her presence.
Question 7 Explanation: 
The correct answer is (C). The speaker, considering a wren she sees above her, states: “What a great beast I must seem, tromping through the mud….”. The speaker is able to consider herself from the wren’s perspective, an idea which is also emphasized in the final stanza. There is no indication that she is uncomfortable with herself (A), or that she chatters (B), or even that she is damaging the forest (D).
Question 8

Differing Views
by Annalise Gulstad

Bubbles squeak out from beneath my feet,
My toes mingle with the mud,
The tiny creek babbles beside me.

Above my head a little brown wren chatters angrily down,
Her chirps of frustration do little along the lines of moving me along,
I breathe out and see her chirping as part of nature’s song.

Pondering her view of the forest creek as compared to mine I consider, What a great beast I must seem, tromping through the mud,
Unlike the deer each of my footfalls lands with a thud.

Meanwhile she flits around between the branches,
Silent but for her song which she raises in fear,
Snatching bugs between the rough bark.

What I had found to be a sweet reprieve,
She perceived as a dangerous siege.

 

The poem ends with the sense of the speaker's

A
joy at being in such a beautiful environment
B
greater understanding of the wren
C
dismay over her destruction of the environment
D
relief to be leaving the forest
Question 8 Explanation: 
The correct answer is (B). The author contrasts her own positive experience in the forest with the alarm she causes the wren. This shows the author has a wider understanding than her own point of view. The author does feel joy at the start of the poem, but this gives way to a humbler, more self-aware tone by the end.
Question 9

Differing Views
by Annalise Gulstad

Bubbles squeak out from beneath my feet,
My toes mingle with the mud,
The tiny creek babbles beside me.

Above my head a little brown wren chatters angrily down,
Her chirps of frustration do little along the lines of moving me along,
I breathe out and see her chirping as part of nature’s song.

Pondering her view of the forest creek as compared to mine I consider, What a great beast I must seem, tromping through the mud,
Unlike the deer each of my footfalls lands with a thud.

Meanwhile she flits around between the branches,
Silent but for her song which she raises in fear,
Snatching bugs between the rough bark.

What I had found to be a sweet reprieve,
She perceived as a dangerous siege.

 

When she considers herself within the scene, the speaker believes she is most like a

A
wren
B
deer
C
beast
D
bug
Question 9 Explanation: 
The correct answer is (C). The speaker calls herself “a great beast,” indicating how she appears to the wren. She does not identify with the wren itself; rather, she emphasizes how she and the wren have different points of view. She also says she is “unlike” the deer, so (B) is incorrect. Bugs are mentioned in the passage, but the speaker does not compare herself to them.
Question 10

Differing Views
by Annalise Gulstad

Bubbles squeak out from beneath my feet,
My toes mingle with the mud,
The tiny creek babbles beside me.

Above my head a little brown wren chatters angrily down,
Her chirps of frustration do little along the lines of moving me along,
I breathe out and see her chirping as part of nature’s song.

Pondering her view of the forest creek as compared to mine I consider, What a great beast I must seem, tromping through the mud,
Unlike the deer each of my footfalls lands with a thud.

Meanwhile she flits around between the branches,
Silent but for her song which she raises in fear,
Snatching bugs between the rough bark.

What I had found to be a sweet reprieve,
She perceived as a dangerous siege.

 

In the last stanza of the poem, the word "reprieve" most nearly means

A
relief
B
affection
C
view
D
revenge
Question 10 Explanation: 
The correct answer is (A). “Reprieve” means temporary relief from something that is uncomfortable or distressing.
Question 11

Differing Views
by Annalise Gulstad

Bubbles squeak out from beneath my feet,
My toes mingle with the mud,
The tiny creek babbles beside me.

Above my head a little brown wren chatters angrily down,
Her chirps of frustration do little along the lines of moving me along,
I breathe out and see her chirping as part of nature’s song.

Pondering her view of the forest creek as compared to mine I consider, What a great beast I must seem, tromping through the mud,
Unlike the deer each of my footfalls lands with a thud.

Meanwhile she flits around between the branches,
Silent but for her song which she raises in fear,
Snatching bugs between the rough bark.

What I had found to be a sweet reprieve,
She perceived as a dangerous siege.

 

Which sentence best describes the organization of the poem?

A
The speaker dedicates each stanza to a separate sense.
B
The speaker makes observations and then forms an opinion.
C
The speaker presents an opinion followed by examples.
D
The speaker describes a process she observes.
Question 11 Explanation: 
The correct answer is (B). In the poem, the speaker first observes how the wren responds to her presence in nature. At the end of the poem, she uses these observations to form an opinion about how the wren must view her.
Question 12
Passage 1

In 1995, environmentalist William Cronon wrote a condemnation of modern environmentalism. Cronon argued that his colleagues idealized the wilderness, forgetting that the wilderness is not a safe place for humans. Either we will tame it and make it no longer wilderness, or we will die within it. And while we are dreaming of the wilderness, we’re not coming up with sustainable ways to live symbiotically within our environment.

In many ways, things have gotten better in the past 25 years, but in many ways they haven’t. Cronon worried about idealism leading to apathy. One might argue that people aren’t as apathetic anymore. Instead of apathy, however, modern environmentalists are fighting willful ignorance. As long as environmentalists have to waste their time re-litigating scientifically-researched and supported facts, we will never be able to slow climate change. The greatest contributors to this willfully ignorant opposition to common sense, unsurprisingly, is the corporations and governments that gain the most from destructive practices. Individuals can recycle all they want, but until corporations take the lead, these individuals are throwing glasses of water at a wildfire.

Passage 2

The problem with environmentalists is that they cry wolf too often. Every five years or so, our globe is on the edge of a crisis. The polar ice caps are supposedly going to melt and destroy everything we hold dear. The hole in the ozone layer is going to give us all skin cancer. The pollution is going to suffocate us. But, you know what? We’re all still here. Every time some environmental “crisis” occurs, the world is suddenly going to end. And yet, the Earth is still spinning.

What are we supposed to do with these empty “warnings,” then? Environmentalists can’t believe that people challenge their claims of man-made climate change. But, we’ve lived through all of these empty claims, these warnings of doom come to naught. For every scientific study that these environmentalists taut as important and true, there are twenty such studies that have proven over time to be false. How, then, can we believe them when they say that climate change is man-made? Or even that climate change exists?
 

The main purpose of the last paragraph of Passage 1 is to

A
provide an update to Cronon’s 25-year-old complaint about environmentalism.
B
refute Cronon’s claim and prove why he was wrong about the wilderness.
C
further support the argument presented in the first paragraph.
D
summarize Cronon’s newest articles on the subject of environmentalism.
Question 12 Explanation: 
Answer choice (A) is correct because after summarizing Cronon’s thoughts from 1995, the author discusses the modern state of environmentalism. This is not a refutation, as suggested in answer choice (B), but just an update on how things have changed over the last 25 years.
Question 13
Passage 1

In 1995, environmentalist William Cronon wrote a condemnation of modern environmentalism. Cronon argued that his colleagues idealized the wilderness, forgetting that the wilderness is not a safe place for humans. Either we will tame it and make it no longer wilderness, or we will die within it. And while we are dreaming of the wilderness, we’re not coming up with sustainable ways to live symbiotically within our environment.

In many ways, things have gotten better in the past 25 years, but in many ways they haven’t. Cronon worried about idealism leading to apathy. One might argue that people aren’t as apathetic anymore. Instead of apathy, however, modern environmentalists are fighting willful ignorance. As long as environmentalists have to waste their time re-litigating scientifically-researched and supported facts, we will never be able to slow climate change. The greatest contributors to this willfully ignorant opposition to common sense, unsurprisingly, is the corporations and governments that gain the most from destructive practices. Individuals can recycle all they want, but until corporations take the lead, these individuals are throwing glasses of water at a wildfire.

Passage 2

The problem with environmentalists is that they cry wolf too often. Every five years or so, our globe is on the edge of a crisis. The polar ice caps are supposedly going to melt and destroy everything we hold dear. The hole in the ozone layer is going to give us all skin cancer. The pollution is going to suffocate us. But, you know what? We’re all still here. Every time some environmental “crisis” occurs, the world is suddenly going to end. And yet, the Earth is still spinning.

What are we supposed to do with these empty “warnings,” then? Environmentalists can’t believe that people challenge their claims of man-made climate change. But, we’ve lived through all of these empty claims, these warnings of doom come to naught. For every scientific study that these environmentalists taut as important and true, there are twenty such studies that have proven over time to be false. How, then, can we believe them when they say that climate change is man-made? Or even that climate change exists?
 

The author of Passage 2 uses the phrase “cry wolf” in order to

A
warn people about the dangers of not taking environmentalism seriously.
B
point out that, like the fairy tale, environmentalists continue to make claims about a danger that doesn’t exist.
C
identify an animal species that needs to be saved from man-made climate change.
D
identify the precise method in which environmentalists lie to the public.
Question 13 Explanation: 
Answer choice (B) is correct because the author is alluding to a fairy tale in which a boy continually makes up stories about a dangerous wolf so that no one listens to him when an actual wolf comes to attack him. The author is suggesting that environmentalists have lied in the past about potential dangers that haven’t come true, so there’s no reason to believe their warnings now.
Question 14
Passage 1

In 1995, environmentalist William Cronon wrote a condemnation of modern environmentalism. Cronon argued that his colleagues idealized the wilderness, forgetting that the wilderness is not a safe place for humans. Either we will tame it and make it no longer wilderness, or we will die within it. And while we are dreaming of the wilderness, we’re not coming up with sustainable ways to live symbiotically within our environment.

In many ways, things have gotten better in the past 25 years, but in many ways they haven’t. Cronon worried about idealism leading to apathy. One might argue that people aren’t as apathetic anymore. Instead of apathy, however, modern environmentalists are fighting willful ignorance. As long as environmentalists have to waste their time re-litigating scientifically-researched and supported facts, we will never be able to slow climate change. The greatest contributors to this willfully ignorant opposition to common sense, unsurprisingly, is the corporations and governments that gain the most from destructive practices. Individuals can recycle all they want, but until corporations take the lead, these individuals are throwing glasses of water at a wildfire.

Passage 2

The problem with environmentalists is that they cry wolf too often. Every five years or so, our globe is on the edge of a crisis. The polar ice caps are supposedly going to melt and destroy everything we hold dear. The hole in the ozone layer is going to give us all skin cancer. The pollution is going to suffocate us. But, you know what? We’re all still here. Every time some environmental “crisis” occurs, the world is suddenly going to end. And yet, the Earth is still spinning.

What are we supposed to do with these empty “warnings,” then? Environmentalists can’t believe that people challenge their claims of man-made climate change. But, we’ve lived through all of these empty claims, these warnings of doom come to naught. For every scientific study that these environmentalists taut as important and true, there are twenty such studies that have proven over time to be false. How, then, can we believe them when they say that climate change is man-made? Or even that climate change exists?
 

Which choice best describes the relationship between the two passages?

A
Passage 1 introduces the idea of environmentalism, and Passage 2 explores it in more detail.
B
Passage 1 is a biased discussion of environmentalism, while Passage 2 remains objective.
C
Passage 1 argues for environmentalism, while Passage 2 questions the validity of the claims made by environmentalists.
D
Passage 2 offers a direct response to the claims made in Passage 1.
Question 14 Explanation: 
The two passages certainly disagree, which eliminates answer choice (A). While they disagree, however, both are biased (B) and neither directly responds to the other (D). The correct answer is answer choice (C).
Question 15
Passage 1

In 1995, environmentalist William Cronon wrote a condemnation of modern environmentalism. Cronon argued that his colleagues idealized the wilderness, forgetting that the wilderness is not a safe place for humans. Either we will tame it and make it no longer wilderness, or we will die within it. And while we are dreaming of the wilderness, we’re not coming up with sustainable ways to live symbiotically within our environment.

In many ways, things have gotten better in the past 25 years, but in many ways they haven’t. Cronon worried about idealism leading to apathy. One might argue that people aren’t as apathetic anymore. Instead of apathy, however, modern environmentalists are fighting willful ignorance. As long as environmentalists have to waste their time re-litigating scientifically-researched and supported facts, we will never be able to slow climate change. The greatest contributors to this willfully ignorant opposition to common sense, unsurprisingly, is the corporations and governments that gain the most from destructive practices. Individuals can recycle all they want, but until corporations take the lead, these individuals are throwing glasses of water at a wildfire.

Passage 2

The problem with environmentalists is that they cry wolf too often. Every five years or so, our globe is on the edge of a crisis. The polar ice caps are supposedly going to melt and destroy everything we hold dear. The hole in the ozone layer is going to give us all skin cancer. The pollution is going to suffocate us. But, you know what? We’re all still here. Every time some environmental “crisis” occurs, the world is suddenly going to end. And yet, the Earth is still spinning.

What are we supposed to do with these empty “warnings,” then? Environmentalists can’t believe that people challenge their claims of man-made climate change. But, we’ve lived through all of these empty claims, these warnings of doom come to naught. For every scientific study that these environmentalists taut as important and true, there are twenty such studies that have proven over time to be false. How, then, can we believe them when they say that climate change is man-made? Or even that climate change exists?
 

Given the evidence in each of the passages, with which statement would the authors of both passages most likely agree?

A
It is clear that humans have had a negative impact on the environment.
B
Environmentalists are often wrong about their claims.
C
William Cronon was right when he suggested that people needed to make realistic changes to try to improve the environment.
D
Some people deny that climate change is affected by humans.
Question 15 Explanation: 
Answer choice (D) is the only option that each author would agree on. Passage 1 discusses people who deny man-made climate change while the author of Passage 2 implies that they are one of those people.
Question 16

Violent Ways

(1) The modern world has a bit of an identity crisis when it comes to their love of sports. On one end of the spectrum, we worry about violence in sports. We’re concerned about head injuries in football. We worry that our children are focusing so early on one sport that they are causing their body harm. We argue that we need to make changes to our current systems to protect our athletes. At the same time, though, violent sports like mixed martial arts have never been more popular. How do we explain such a split personality in how we view athletes? Why are we worried about a quarterback getting hit too hard but cheering when a fighter lifelessly collapses to the mat like a dead tree?

(2) The answer to these questions is at the same time simple and complicated. The simple answer is that a sport like mixed martial arts is intended to be violent; the sport involves two people getting into a ring and fighting. Football, on the other hand, centers around moving a ball from one zone to the next. To argue that football is not about violence would be extremely naïve, though. That’s where the problem gets complicated. If we remove violence from football, what’s left?

(3) The more complicated and depressing reason why we can’t seem to figure out whether we want violence or not is because it plays to our human nature. The football arena is the new Colosseum, and the players are the new gladiators. One of the most powerful human instinct is the instinct of survival. Naturally, then, we are entertained when people are in peril because we are instinctively drawn to their fight for survival. This doesn’t make us evil; we don’t want to see people get hurt. We do, however, want to see people get hit.

(4) The problem is that there is no definitive solution to violence in sports. We want it and don’t want it at the same time. It seems our culture has decided to allow it in certain areas where we think it belongs, like mixed martial arts, but condemning it where we think it doesn’t, like football. Football fans worry about what’s going to happen to the sport in the next few years as reports of brain injuries continue to surface. Most likely, though, nothing will change. We will sit around our dinner table and argue that something needs to be done, then head to the Colosseum and cheer for every big hit.
 

Read this sentence from the passage:

The football arena is the new Colosseum, and the players are the new gladiators.
 

Which of these best explains the meaning of the phrase “the players are the new gladiators”?

A
Football is an outdated game that needs to fall just like the empire of Rome fell.
B
Football players are violent and want to hurt each other.
C
People like to see football players hit each other in the same way that people loved to watch gladiators kill themselves in ancient Rome.
D
The more we watch football players, the more we give in to our instincts and cause harm to our society.
Question 16 Explanation: 
The author alludes to ancient Rome in this quote in order to try to explain our obsession with violence. Answer choice (C) best captures this meaning and tone. The author isn’t condemning football player (B), or calling for the end of football (A) (D), she is instead offering an insight into why Americans like it.
Question 17

Violent Ways

(1) The modern world has a bit of an identity crisis when it comes to their love of sports. On one end of the spectrum, we worry about violence in sports. We’re concerned about head injuries in football. We worry that our children are focusing so early on one sport that they are causing their body harm. We argue that we need to make changes to our current systems to protect our athletes. At the same time, though, violent sports like mixed martial arts have never been more popular. How do we explain such a split personality in how we view athletes? Why are we worried about a quarterback getting hit too hard but cheering when a fighter lifelessly collapses to the mat like a dead tree?

(2) The answer to these questions is at the same time simple and complicated. The simple answer is that a sport like mixed martial arts is intended to be violent; the sport involves two people getting into a ring and fighting. Football, on the other hand, centers around moving a ball from one zone to the next. To argue that football is not about violence would be extremely naïve, though. That’s where the problem gets complicated. If we remove violence from football, what’s left?

(3) The more complicated and depressing reason why we can’t seem to figure out whether we want violence or not is because it plays to our human nature. The football arena is the new Colosseum, and the players are the new gladiators. One of the most powerful human instinct is the instinct of survival. Naturally, then, we are entertained when people are in peril because we are instinctively drawn to their fight for survival. This doesn’t make us evil; we don’t want to see people get hurt. We do, however, want to see people get hit.

(4) The problem is that there is no definitive solution to violence in sports. We want it and don’t want it at the same time. It seems our culture has decided to allow it in certain areas where we think it belongs, like mixed martial arts, but condemning it where we think it doesn’t, like football. Football fans worry about what’s going to happen to the sport in the next few years as reports of brain injuries continue to surface. Most likely, though, nothing will change. We will sit around our dinner table and argue that something needs to be done, then head to the Colosseum and cheer for every big hit.
 

How does Paragraph 2 contribute to the development of ideas in the passage?

A
It establishes the complexity of the topic the author is discussing.
B
It simplifies the topic to make it more palatable and digestible for the audience.
C
It presents a counter-argument that she will refute later in the article.
D
It defines the author’s thesis and sets up the rest of the article.
Question 17 Explanation: 
Answer choice (A) is correct because even though the author initially offers a simple answer to the question she posed in the previous paragraph, she also takes that answer apart and prepares the audience for the complexities she will detail throughout the rest of the argument. This isn’t necessarily a counter-argument (C), nor does it set up the author’s thesis (D). She presents her thesis in Paragraph 3.
Question 18

Violent Ways

(1) The modern world has a bit of an identity crisis when it comes to their love of sports. On one end of the spectrum, we worry about violence in sports. We’re concerned about head injuries in football. We worry that our children are focusing so early on one sport that they are causing their body harm. We argue that we need to make changes to our current systems to protect our athletes. At the same time, though, violent sports like mixed martial arts have never been more popular. How do we explain such a split personality in how we view athletes? Why are we worried about a quarterback getting hit too hard but cheering when a fighter lifelessly collapses to the mat like a dead tree?

(2) The answer to these questions is at the same time simple and complicated. The simple answer is that a sport like mixed martial arts is intended to be violent; the sport involves two people getting into a ring and fighting. Football, on the other hand, centers around moving a ball from one zone to the next. To argue that football is not about violence would be extremely naïve, though. That’s where the problem gets complicated. If we remove violence from football, what’s left?

(3) The more complicated and depressing reason why we can’t seem to figure out whether we want violence or not is because it plays to our human nature. The football arena is the new Colosseum, and the players are the new gladiators. One of the most powerful human instinct is the instinct of survival. Naturally, then, we are entertained when people are in peril because we are instinctively drawn to their fight for survival. This doesn’t make us evil; we don’t want to see people get hurt. We do, however, want to see people get hit.

(4) The problem is that there is no definitive solution to violence in sports. We want it and don’t want it at the same time. It seems our culture has decided to allow it in certain areas where we think it belongs, like mixed martial arts, but condemning it where we think it doesn’t, like football. Football fans worry about what’s going to happen to the sport in the next few years as reports of brain injuries continue to surface. Most likely, though, nothing will change. We will sit around our dinner table and argue that something needs to be done, then head to the Colosseum and cheer for every big hit.
 

Read this sentence from Paragraph 1:

The modern world has a bit of an identity crisis when it comes to their love of sports.
 

Based on the context of the passage, what is the problem (if any) with this quote?

A
A culture cannot have an “identity crisis” because it is not a living thing.
B
The author introduces the “world” in this quote, but only discusses American culture throughout the article.
C
The word “bit” is too informal for the article and distracts the reader.
D
There is no flaw in the logic of the statement.
Question 18 Explanation: 
The author introduces the “world” in this quote, but then discusses mainly football and mixed martial arts, which are both mostly American sports. If the author intends to discuss the state of the world in the article, then she has alienated a lot of her audience by only discussing things that Americans can relate to.
Question 19

Violent Ways

(1) The modern world has a bit of an identity crisis when it comes to their love of sports. On one end of the spectrum, we worry about violence in sports. We’re concerned about head injuries in football. We worry that our children are focusing so early on one sport that they are causing their body harm. We argue that we need to make changes to our current systems to protect our athletes. At the same time, though, violent sports like mixed martial arts have never been more popular. How do we explain such a split personality in how we view athletes? Why are we worried about a quarterback getting hit too hard but cheering when a fighter lifelessly collapses to the mat like a dead tree?

(2) The answer to these questions is at the same time simple and complicated. The simple answer is that a sport like mixed martial arts is intended to be violent; the sport involves two people getting into a ring and fighting. Football, on the other hand, centers around moving a ball from one zone to the next. To argue that football is not about violence would be extremely naïve, though. That’s where the problem gets complicated. If we remove violence from football, what’s left?

(3) The more complicated and depressing reason why we can’t seem to figure out whether we want violence or not is because it plays to our human nature. The football arena is the new Colosseum, and the players are the new gladiators. One of the most powerful human instinct is the instinct of survival. Naturally, then, we are entertained when people are in peril because we are instinctively drawn to their fight for survival. This doesn’t make us evil; we don’t want to see people get hurt. We do, however, want to see people get hit.

(4) The problem is that there is no definitive solution to violence in sports. We want it and don’t want it at the same time. It seems our culture has decided to allow it in certain areas where we think it belongs, like mixed martial arts, but condemning it where we think it doesn’t, like football. Football fans worry about what’s going to happen to the sport in the next few years as reports of brain injuries continue to surface. Most likely, though, nothing will change. We will sit around our dinner table and argue that something needs to be done, then head to the Colosseum and cheer for every big hit.
 

Which sentence from the passage best expresses the author’s attitude toward violence in sports?

A
“The simple answer is that a sport like mixed martial arts is intended to be violent; the sport involves two people getting into a ring and fighting.”
B
“We worry that our children are focusing so early on one sport that they are causing their body harm.”
C
“Football fans worry about what’s going to happen to the sport in the next few years as reports of brain injuries continue to surface.”
D
“The more complicated and depressing reason why we can’t seem to figure out whether we want violence or not is because it plays to our human nature.”
Question 19 Explanation: 
Answer choice (D) is correct because it is the only quote that discusses the author’s explanation for the world’s obsession with violence. The other answer choices represent ideas that the author uses to build to her thesis, not directly express her attitude towards the subject.
Question 20

Violent Ways

(1) The modern world has a bit of an identity crisis when it comes to their love of sports. On one end of the spectrum, we worry about violence in sports. We’re concerned about head injuries in football. We worry that our children are focusing so early on one sport that they are causing their body harm. We argue that we need to make changes to our current systems to protect our athletes. At the same time, though, violent sports like mixed martial arts have never been more popular. How do we explain such a split personality in how we view athletes? Why are we worried about a quarterback getting hit too hard but cheering when a fighter lifelessly collapses to the mat like a dead tree?

(2) The answer to these questions is at the same time simple and complicated. The simple answer is that a sport like mixed martial arts is intended to be violent; the sport involves two people getting into a ring and fighting. Football, on the other hand, centers around moving a ball from one zone to the next. To argue that football is not about violence would be extremely naïve, though. That’s where the problem gets complicated. If we remove violence from football, what’s left?

(3) The more complicated and depressing reason why we can’t seem to figure out whether we want violence or not is because it plays to our human nature. The football arena is the new Colosseum, and the players are the new gladiators. One of the most powerful human instinct is the instinct of survival. Naturally, then, we are entertained when people are in peril because we are instinctively drawn to their fight for survival. This doesn’t make us evil; we don’t want to see people get hurt. We do, however, want to see people get hit.

(4) The problem is that there is no definitive solution to violence in sports. We want it and don’t want it at the same time. It seems our culture has decided to allow it in certain areas where we think it belongs, like mixed martial arts, but condemning it where we think it doesn’t, like football. Football fans worry about what’s going to happen to the sport in the next few years as reports of brain injuries continue to surface. Most likely, though, nothing will change. We will sit around our dinner table and argue that something needs to be done, then head to the Colosseum and cheer for every big hit.
 

Which of the following is an inference the audience can make about the author’s opinion in Paragraph 4?

A
When people think rationally, they don’t want violence, but when they are in the moment they enjoy it.
B
Until we stop supporting football, our country will never fix our problem with violence.
C
The popularity of violent sports like mixed martial arts are making it more difficult to curb violence in other sports.
D
The real way to fix our problem with violence in sports is to more closely regulate it so that our athletes are protected.
Question 20 Explanation: 
Answer choice (A) is correct because it is a more specific explanation of the author’s closing line. The author believes that people want to be against violence, but that they can’t help but be entertained by it when they see it. Answer choice (B) and (C) are not reflected in the article, and answer choice (D) goes against the opening sentence of Paragraph 4.
Question 21

Violent Ways

(1) The modern world has a bit of an identity crisis when it comes to their love of sports. On one end of the spectrum, we worry about violence in sports. We’re concerned about head injuries in football. We worry that our children are focusing so early on one sport that they are causing their body harm. We argue that we need to make changes to our current systems to protect our athletes. At the same time, though, violent sports like mixed martial arts have never been more popular. How do we explain such a split personality in how we view athletes? Why are we worried about a quarterback getting hit too hard but cheering when a fighter lifelessly collapses to the mat like a dead tree?

(2) The answer to these questions is at the same time simple and complicated. The simple answer is that a sport like mixed martial arts is intended to be violent; the sport involves two people getting into a ring and fighting. Football, on the other hand, centers around moving a ball from one zone to the next. To argue that football is not about violence would be extremely naïve, though. That’s where the problem gets complicated. If we remove violence from football, what’s left?

(3) The more complicated and depressing reason why we can’t seem to figure out whether we want violence or not is because it plays to our human nature. The football arena is the new Colosseum, and the players are the new gladiators. One of the most powerful human instinct is the instinct of survival. Naturally, then, we are entertained when people are in peril because we are instinctively drawn to their fight for survival. This doesn’t make us evil; we don’t want to see people get hurt. We do, however, want to see people get hit.

(4) The problem is that there is no definitive solution to violence in sports. We want it and don’t want it at the same time. It seems our culture has decided to allow it in certain areas where we think it belongs, like mixed martial arts, but condemning it where we think it doesn’t, like football. Football fans worry about what’s going to happen to the sport in the next few years as reports of brain injuries continue to surface. Most likely, though, nothing will change. We will sit around our dinner table and argue that something needs to be done, then head to the Colosseum and cheer for every big hit.
 

Which of the following statements best describes the structure of the author’s article?

A
The author introduces a complex problem, then offers several possible solutions.
B
The author presents two opposing sides to a complex argument and leaves it up to the reader decide with which side they agree.
C
The author provides a detailed, sequential strategy for solving a difficult problem.
D
The author introduces a complex problem, then explores it throughout the rest of the article.
Question 21 Explanation: 
The author offers plenty of opinions throughout the article, but never ventures into a real solution to the problem. Answer choice (D) is the only option that allows for this lack of a solution.
Question 22

Violent Ways

(1) The modern world has a bit of an identity crisis when it comes to their love of sports. On one end of the spectrum, we worry about violence in sports. We’re concerned about head injuries in football. We worry that our children are focusing so early on one sport that they are causing their body harm. We argue that we need to make changes to our current systems to protect our athletes. At the same time, though, violent sports like mixed martial arts have never been more popular. How do we explain such a split personality in how we view athletes? Why are we worried about a quarterback getting hit too hard but cheering when a fighter lifelessly collapses to the mat like a dead tree?

(2) The answer to these questions is at the same time simple and complicated. The simple answer is that a sport like mixed martial arts is intended to be violent; the sport involves two people getting into a ring and fighting. Football, on the other hand, centers around moving a ball from one zone to the next. To argue that football is not about violence would be extremely naïve, though. That’s where the problem gets complicated. If we remove violence from football, what’s left?

(3) The more complicated and depressing reason why we can’t seem to figure out whether we want violence or not is because it plays to our human nature. The football arena is the new Colosseum, and the players are the new gladiators. One of the most powerful human instinct is the instinct of survival. Naturally, then, we are entertained when people are in peril because we are instinctively drawn to their fight for survival. This doesn’t make us evil; we don’t want to see people get hurt. We do, however, want to see people get hit.

(4) The problem is that there is no definitive solution to violence in sports. We want it and don’t want it at the same time. It seems our culture has decided to allow it in certain areas where we think it belongs, like mixed martial arts, but condemning it where we think it doesn’t, like football. Football fans worry about what’s going to happen to the sport in the next few years as reports of brain injuries continue to surface. Most likely, though, nothing will change. We will sit around our dinner table and argue that something needs to be done, then head to the Colosseum and cheer for every big hit.
 

Read this sentence from Paragraph 4:

We will sit around our dinner table and argue that something needs to be done, then head to the Colosseum and cheer for every big hit.
 

Why does the author use the phrase “dinner table” in this sentence?

A
To stress the idea that people only find violence a problem in sports when they are trying to eat a meal and don’t want that kind of imagery in their minds.
B
To point that people don’t actually share their true feelings at the “dinner table” because they are afraid of what their family will think.
C
To emphasize the idea that people are anti-violent sports when they are discussing the problem at home at the “dinner table,” but act differently when they are watching a sporting event.
D
To highlight the point that violence in sports has gotten so prevalent that it’s even invading our time with our family at home.
Question 22 Explanation: 
The author is using the “dinner table” as juxtaposition against “the Colosseum” to emphasize her point that we may be against violent sports in theory, but entertained by it in practice. None of the other answer choices really address this comparison.
Question 23

Inaugural Address of John F. Kennedy

So let us begin a new remembering on both sides that civility is not a sign of weakness, and sincerity is always subject to proof. Let us never negotiate out of fear. But let us never fear to negotiate. And if a beachhead of cooperation may push back the jungle of suspicion, let both sides join in creating a new endeavor, not a new balance of power, but a new world of law, where the strong are just and the weak secure and the peace preserved.

All this will not be finished in the first one hundred days. Nor will it be finished in the first one thousand days, nor in the life of this Administration, nor even perhaps in our lifetime on this planet. But let us begin.

In your hands, my fellow citizens, more than mine, will rest the final success or failure of our course. Since this country was founded, each generation of Americans has been summoned to give testimony to its national loyalty. The graves of young Americans who answered the call to service surround the globe.

Now the trumpet summons us again; not as a call to bear arms, though arms we need; not as a call to battle, though embattled we are; but a call to bear the burden of a long twilight struggle, year in and year out, "rejoicing in hope, patient in tribulation"—a struggle against the common enemies of man: tyranny, poverty, disease and war itself. Can we forge against these enemies a grand and global alliance, North and South, East and West, that can assure a more fruitful life for all mankind? Will you join in that historic effort?

In the long history of the world, only a few generations have been granted the role of defending freedom in its hour of maximum danger. I do not shrink from this responsibility; I welcome it. I do not believe that any of us would exchange places with any other people or any other generation. The energy, the faith, the devotion which we bring to this endeavor will light our country and all who serve it—and the glow from that fire can truly light the world.

And so, my fellow Americans: ask not what your country can do for you—ask what you can do for your country. My fellow citizens of the world: ask not what America will do for you, but what together we can do for the freedom of man.

Finally, whether you are citizens of America or citizens of the world, ask of us here the same high standards of strength and sacrifice which we ask of you. With a good conscience our only sure reward, with history the final judge of our deeds, let us go forth to lead the land we love, asking His blessing and His help, but knowing that here on Earth God's work must truly be our own.
 

What is President Kennedy’s main purpose in this address?

A
to encourage Americans to be more civil with each other and with America’s foreign allies
B
to ask Congress to give the executive branch more power in military affairs
C
to illustrate the precarious position of America in foreign affairs
D
to rally Americans to serve their country
Question 23 Explanation: 
The correct answer is (D). Kennedy’s main purpose is best encapsulated in the sentence, “And so, my fellow Americans: ask not what your country can do for you—ask what you can do for your country.” Kennedy is urging Americans to rise up and promote freedom domestically and abroad.
Question 24

Inaugural Address of John F. Kennedy

So let us begin a new remembering on both sides that civility is not a sign of weakness, and sincerity is always subject to proof. Let us never negotiate out of fear. But let us never fear to negotiate. And if a beachhead of cooperation may push back the jungle of suspicion, let both sides join in creating a new endeavor, not a new balance of power, but a new world of law, where the strong are just and the weak secure and the peace preserved.

All this will not be finished in the first one hundred days. Nor will it be finished in the first one thousand days, nor in the life of this Administration, nor even perhaps in our lifetime on this planet. But let us begin.

In your hands, my fellow citizens, more than mine, will rest the final success or failure of our course. Since this country was founded, each generation of Americans has been summoned to give testimony to its national loyalty. The graves of young Americans who answered the call to service surround the globe.

Now the trumpet summons us again; not as a call to bear arms, though arms we need; not as a call to battle, though embattled we are; but a call to bear the burden of a long twilight struggle, year in and year out, "rejoicing in hope, patient in tribulation"—a struggle against the common enemies of man: tyranny, poverty, disease and war itself. Can we forge against these enemies a grand and global alliance, North and South, East and West, that can assure a more fruitful life for all mankind? Will you join in that historic effort?

In the long history of the world, only a few generations have been granted the role of defending freedom in its hour of maximum danger. I do not shrink from this responsibility; I welcome it. I do not believe that any of us would exchange places with any other people or any other generation. The energy, the faith, the devotion which we bring to this endeavor will light our country and all who serve it—and the glow from that fire can truly light the world.

And so, my fellow Americans: ask not what your country can do for you—ask what you can do for your country. My fellow citizens of the world: ask not what America will do for you, but what together we can do for the freedom of man.

Finally, whether you are citizens of America or citizens of the world, ask of us here the same high standards of strength and sacrifice which we ask of you. With a good conscience our only sure reward, with history the final judge of our deeds, let us go forth to lead the land we love, asking His blessing and His help, but knowing that here on Earth God's work must truly be our own.
 

In the final paragraph, what hope does President Kennedy express about his government?

A
that it will be better than the preceding government
B
that it will be held accountable by the people
C
that it will make future generations proud
D
that it will be ruled by God’s will
Question 24 Explanation: 
The correct answer is (B). In the final paragraph, Kennedy states, “ask of us here the same high standards of strength and sacrifice which we ask of you.” He encourages the people to hold his government accountable. Nowhere in the final paragraph does Kennedy mention the preceding government or future generation. Answers (C) and (D) are also unsupported by the final paragraph. God is mentioned, but Kennedy does not express a hope that his government will be ruled by God’s will.
Question 25

Inaugural Address of John F. Kennedy

So let us begin a new remembering on both sides that civility is not a sign of weakness, and sincerity is always subject to proof. Let us never negotiate out of fear. But let us never fear to negotiate. And if a beachhead of cooperation may push back the jungle of suspicion, let both sides join in creating a new endeavor, not a new balance of power, but a new world of law, where the strong are just and the weak secure and the peace preserved.

All this will not be finished in the first one hundred days. Nor will it be finished in the first one thousand days, nor in the life of this Administration, nor even perhaps in our lifetime on this planet. But let us begin.

In your hands, my fellow citizens, more than mine, will rest the final success or failure of our course. Since this country was founded, each generation of Americans has been summoned to give testimony to its national loyalty. The graves of young Americans who answered the call to service surround the globe.

Now the trumpet summons us again; not as a call to bear arms, though arms we need; not as a call to battle, though embattled we are; but a call to bear the burden of a long twilight struggle, year in and year out, "rejoicing in hope, patient in tribulation"—a struggle against the common enemies of man: tyranny, poverty, disease and war itself. Can we forge against these enemies a grand and global alliance, North and South, East and West, that can assure a more fruitful life for all mankind? Will you join in that historic effort?

In the long history of the world, only a few generations have been granted the role of defending freedom in its hour of maximum danger. I do not shrink from this responsibility; I welcome it. I do not believe that any of us would exchange places with any other people or any other generation. The energy, the faith, the devotion which we bring to this endeavor will light our country and all who serve it—and the glow from that fire can truly light the world.

And so, my fellow Americans: ask not what your country can do for you—ask what you can do for your country. My fellow citizens of the world: ask not what America will do for you, but what together we can do for the freedom of man.

Finally, whether you are citizens of America or citizens of the world, ask of us here the same high standards of strength and sacrifice which we ask of you. With a good conscience our only sure reward, with history the final judge of our deeds, let us go forth to lead the land we love, asking His blessing and His help, but knowing that here on Earth God's work must truly be our own.
 

What is “the trumpet” a metaphor for in the beginning of the fourth paragraph?

A
a call to bear arms and fight foreign dictatorships
B
a call to overcome domestic strife and petty differences
C
a call to build up the military and allocate more tax revenue to that purpose
D
a call to endure the challenges that come with fighting tyranny
Question 25 Explanation: 
The correct answer is (D). Immediately after mentioning the “trumpet,” Kennedy insists it is NOT a call to bear arms, so (A) and (C) are not logical answers. The scope of the paragraph, however, is international as well as national, so the “trumpet” is beyond the “domestic” concerns mentioned in choice (B). Choice (D) is the best rephrase of the information in the paragraph.
Question 26

Inaugural Address of John F. Kennedy

So let us begin a new remembering on both sides that civility is not a sign of weakness, and sincerity is always subject to proof. Let us never negotiate out of fear. But let us never fear to negotiate. And if a beachhead of cooperation may push back the jungle of suspicion, let both sides join in creating a new endeavor, not a new balance of power, but a new world of law, where the strong are just and the weak secure and the peace preserved.

All this will not be finished in the first one hundred days. Nor will it be finished in the first one thousand days, nor in the life of this Administration, nor even perhaps in our lifetime on this planet. But let us begin.

In your hands, my fellow citizens, more than mine, will rest the final success or failure of our course. Since this country was founded, each generation of Americans has been summoned to give testimony to its national loyalty. The graves of young Americans who answered the call to service surround the globe.

Now the trumpet summons us again; not as a call to bear arms, though arms we need; not as a call to battle, though embattled we are; but a call to bear the burden of a long twilight struggle, year in and year out, "rejoicing in hope, patient in tribulation"—a struggle against the common enemies of man: tyranny, poverty, disease and war itself. Can we forge against these enemies a grand and global alliance, North and South, East and West, that can assure a more fruitful life for all mankind? Will you join in that historic effort?

In the long history of the world, only a few generations have been granted the role of defending freedom in its hour of maximum danger. I do not shrink from this responsibility; I welcome it. I do not believe that any of us would exchange places with any other people or any other generation. The energy, the faith, the devotion which we bring to this endeavor will light our country and all who serve it—and the glow from that fire can truly light the world.

And so, my fellow Americans: ask not what your country can do for you—ask what you can do for your country. My fellow citizens of the world: ask not what America will do for you, but what together we can do for the freedom of man.

Finally, whether you are citizens of America or citizens of the world, ask of us here the same high standards of strength and sacrifice which we ask of you. With a good conscience our only sure reward, with history the final judge of our deeds, let us go forth to lead the land we love, asking His blessing and His help, but knowing that here on Earth God's work must truly be our own.
 

How does Kennedy attempt to recruit Americans to join the fight against tyranny?

A
by placing it in a historical context
B
by boasting of his personal bravery and confidence, and encouraging American pride
C
by outlining a plan of action for future American conflicts
D
by criticizing the inaction of other countries
Question 26 Explanation: 
The correct answer is (A). Kennedy inspires his audience by placing the fight against tyranny in a global, historical, and immediate context. He does not boast of his own experiences, he does not outline a specific plan of action regarding future conflicts, nor does he criticize the inaction of other countries. In paragraph 3 he places the success of “our course” in the hands of the people and puts the situation in perspective by comparing the current situation with that of earlier generations. By doing so, Kennedy introduces a historical argument meant to encourage the people to continue working toward success.
Question 27

Inaugural Address of John F. Kennedy

So let us begin a new remembering on both sides that civility is not a sign of weakness, and sincerity is always subject to proof. Let us never negotiate out of fear. But let us never fear to negotiate. And if a beachhead of cooperation may push back the jungle of suspicion, let both sides join in creating a new endeavor, not a new balance of power, but a new world of law, where the strong are just and the weak secure and the peace preserved.

All this will not be finished in the first one hundred days. Nor will it be finished in the first one thousand days, nor in the life of this Administration, nor even perhaps in our lifetime on this planet. But let us begin.

In your hands, my fellow citizens, more than mine, will rest the final success or failure of our course. Since this country was founded, each generation of Americans has been summoned to give testimony to its national loyalty. The graves of young Americans who answered the call to service surround the globe.

Now the trumpet summons us again; not as a call to bear arms, though arms we need; not as a call to battle, though embattled we are; but a call to bear the burden of a long twilight struggle, year in and year out, "rejoicing in hope, patient in tribulation"—a struggle against the common enemies of man: tyranny, poverty, disease and war itself. Can we forge against these enemies a grand and global alliance, North and South, East and West, that can assure a more fruitful life for all mankind? Will you join in that historic effort?

In the long history of the world, only a few generations have been granted the role of defending freedom in its hour of maximum danger. I do not shrink from this responsibility; I welcome it. I do not believe that any of us would exchange places with any other people or any other generation. The energy, the faith, the devotion which we bring to this endeavor will light our country and all who serve it—and the glow from that fire can truly light the world.

And so, my fellow Americans: ask not what your country can do for you—ask what you can do for your country. My fellow citizens of the world: ask not what America will do for you, but what together we can do for the freedom of man.

Finally, whether you are citizens of America or citizens of the world, ask of us here the same high standards of strength and sacrifice which we ask of you. With a good conscience our only sure reward, with history the final judge of our deeds, let us go forth to lead the land we love, asking His blessing and His help, but knowing that here on Earth God's work must truly be our own.
 

Which of these statements most strongly supports your response to the previous question?

(Use the left arrow below to go back and review the previous question.)

A
In your hands, my fellow citizens, more than mine, will rest the final success or failure of our course.
B
In the long history of the world, only a few generations have been granted the role of defending freedom in its hour of maximum danger.
C
I do not shrink from this responsibility; I welcome it.
D
Finally, whether you are citizens of America or citizens of the world, ask of us here the same high standards of strength and sacrifice which we ask of you.
Question 27 Explanation: 
The correct answer is (B). The only statement here that places the conflict in a historical context is choice (B), since it discusses the “long history of the world,” and implicitly compares the circumstances of the current generation with that of previous generations.
Question 28

Dolphin Communication

The study of cetacean intelligence has added to our understanding of dolphin behavior. Even though cetaceans—marine animals including dolphins, whales, and porpoises—are widely considered intelligent species, conclusions about the type and extent of dolphin intelligence have not yet been reached. However, there are several things that we do know about the ways in which dolphins communicate and behave in groups.

Researchers study the level of communication among dolphins because communication systems can offer insight into an animal’s intelligence. Dolphins produce two primary types of vocalizations called clicks and whistles. Dolphins generally use clicks for the purposes of echolocation and whistles for communication. Dolphins emit clicks as rapid broadband bursts that are sent out into their environment and then echo back, giving them information about their surroundings. Whistles differ from clicks by being sent out as narrow-band frequency modulated signals, which are for communications such as contact calls. Strong evidence supports the idea that dolphins use signature whistles to identify and call to each other.

One hypothesis called the “acoustic flashlight” hypothesis maintains that dolphins may be able to learn passively about their surroundings by listening in on the echolocative inspections of other dolphins. Scientists are researching this idea by looking at how dolphins participate in postural pointing, an activity that humans perform when they point a finger to direct another person towards an object or location.

Communication is also an important part of how dolphins behave in groups, which can range in size from a pair of dolphins to a pod of hundreds of individuals. Researchers have noticed that large packs rely on a highly organized method of communication when responding to predators, such as sharks. When a group of dolphins reacts to an unexpected disturbance, it will quickly move in near-unison to avoid the threat. In this way, dolphins rely on visual and auditory cues to communicate their location and to interpret the locations of others in the pod.
 

It can be inferred from the passage that a dolphin would use clicks instead of whistles to

A
call to another member of the same pod.
B
locate a possible route through an underwater obstacle.
C
communicate a perceived threat to other dolphins.
D
contact other dolphins when separated from a group.
Question 28 Explanation: 
The correct answer is (B). Paragraph two discusses how dolphins use clicks and whistles. Notice that the passage states that dolphins "generally use clicks for the purposes of echolocation and whistles for communication." If dolphins use clicks for echolocation, we can infer they would use them to “locate a possible route through an underwater obstacle.” Nether of the other answer choices represent situation where a dolphin would use echolocation.
Question 29

Dolphin Communication

The study of cetacean intelligence has added to our understanding of dolphin behavior. Even though cetaceans—marine animals including dolphins, whales, and porpoises—are widely considered intelligent species, conclusions about the type and extent of dolphin intelligence have not yet been reached. However, there are several things that we do know about the ways in which dolphins communicate and behave in groups.

Researchers study the level of communication among dolphins because communication systems can offer insight into an animal’s intelligence. Dolphins produce two primary types of vocalizations called clicks and whistles. Dolphins generally use clicks for the purposes of echolocation and whistles for communication. Dolphins emit clicks as rapid broadband bursts that are sent out into their environment and then echo back, giving them information about their surroundings. Whistles differ from clicks by being sent out as narrow-band frequency modulated signals, which are for communications such as contact calls. Strong evidence supports the idea that dolphins use signature whistles to identify and call to each other.

One hypothesis called the “acoustic flashlight” hypothesis maintains that dolphins may be able to learn passively about their surroundings by listening in on the echolocative inspections of other dolphins. Scientists are researching this idea by looking at how dolphins participate in postural pointing, an activity that humans perform when they point a finger to direct another person towards an object or location.

Communication is also an important part of how dolphins behave in groups, which can range in size from a pair of dolphins to a pod of hundreds of individuals. Researchers have noticed that large packs rely on a highly organized method of communication when responding to predators, such as sharks. When a group of dolphins reacts to an unexpected disturbance, it will quickly move in near-unison to avoid the threat. In this way, dolphins rely on visual and auditory cues to communicate their location and to interpret the locations of others in the pod.
 

According to the passage, highly organized communication enables dolphins to

A
work in unison to locate food sources.
B
move quickly as a group to escape a perceived threat.
C
establish relationships quickly with their peers.
D
have more highly organized hunting patterns.
Question 29 Explanation: 
The correct answer is (B). The phrase "according to the passage" provides a clue to look directly to the passage for the answer. Notice that paragraph four discusses an example when dolphins move "quickly in near-unison to avoid a threat." This example explains how dolphins use highly organized communication to move as a group to escape.
Question 30

Dolphin Communication

The study of cetacean intelligence has added to our understanding of dolphin behavior. Even though cetaceans—marine animals including dolphins, whales, and porpoises—are widely considered intelligent species, conclusions about the type and extent of dolphin intelligence have not yet been reached. However, there are several things that we do know about the ways in which dolphins communicate and behave in groups.

Researchers study the level of communication among dolphins because communication systems can offer insight into an animal’s intelligence. Dolphins produce two primary types of vocalizations called clicks and whistles. Dolphins generally use clicks for the purposes of echolocation and whistles for communication. Dolphins emit clicks as rapid broadband bursts that are sent out into their environment and then echo back, giving them information about their surroundings. Whistles differ from clicks by being sent out as narrow-band frequency modulated signals, which are for communications such as contact calls. Strong evidence supports the idea that dolphins use signature whistles to identify and call to each other.

One hypothesis called the “acoustic flashlight” hypothesis maintains that dolphins may be able to learn passively about their surroundings by listening in on the echolocative inspections of other dolphins. Scientists are researching this idea by looking at how dolphins participate in postural pointing, an activity that humans perform when they point a finger to direct another person towards an object or location.

Communication is also an important part of how dolphins behave in groups, which can range in size from a pair of dolphins to a pod of hundreds of individuals. Researchers have noticed that large packs rely on a highly organized method of communication when responding to predators, such as sharks. When a group of dolphins reacts to an unexpected disturbance, it will quickly move in near-unison to avoid the threat. In this way, dolphins rely on visual and auditory cues to communicate their location and to interpret the locations of others in the pod.
 

As it is used in the second paragraph, the word “emit” most nearly means

A
secrete
B
radiate
C
produce
D
spout
Question 30 Explanation: 
The correct answer is (C). When the sentence says “Dolphins emit clicks,” the word means they produce or make the sound. The other answers provide definitions for the word “emit” that do not fit the context.
Question 31

Dolphin Communication

The study of cetacean intelligence has added to our understanding of dolphin behavior. Even though cetaceans—marine animals including dolphins, whales, and porpoises—are widely considered intelligent species, conclusions about the type and extent of dolphin intelligence have not yet been reached. However, there are several things that we do know about the ways in which dolphins communicate and behave in groups.

Researchers study the level of communication among dolphins because communication systems can offer insight into an animal’s intelligence. Dolphins produce two primary types of vocalizations called clicks and whistles. Dolphins generally use clicks for the purposes of echolocation and whistles for communication. Dolphins emit clicks as rapid broadband bursts that are sent out into their environment and then echo back, giving them information about their surroundings. Whistles differ from clicks by being sent out as narrow-band frequency modulated signals, which are for communications such as contact calls. Strong evidence supports the idea that dolphins use signature whistles to identify and call to each other.

One hypothesis called the “acoustic flashlight” hypothesis maintains that dolphins may be able to learn passively about their surroundings by listening in on the echolocative inspections of other dolphins. Scientists are researching this idea by looking at how dolphins participate in postural pointing, an activity that humans perform when they point a finger to direct another person towards an object or location.

Communication is also an important part of how dolphins behave in groups, which can range in size from a pair of dolphins to a pod of hundreds of individuals. Researchers have noticed that large packs rely on a highly organized method of communication when responding to predators, such as sharks. When a group of dolphins reacts to an unexpected disturbance, it will quickly move in near-unison to avoid the threat. In this way, dolphins rely on visual and auditory cues to communicate their location and to interpret the locations of others in the pod.
 

Which of the following is the function of the first paragraph?

A
to contrast the communication styles of dolphins with those of other cetaceans
B
to define an abstract idea in order to simplify the discussion that follows
C
to introduce the passage’s focus on how the dolphins communicate and behave in groups
D
to show the consequences of a mistake made in earlier research
Question 31 Explanation: 
The correct answer is (C). This question is testing your ability to determine how one part of the passage relates to the whole. In general, the first paragraph of most essays will introduce the main idea. The first paragraph introduces the topic of the passage, namely that "there are several things that we do know about the ways in which dolphins communicate and behave in groups." The remainder of the passage discusses dolphin communication and behavior. While the first paragraph does mention other cetaceans, it does not discuss their communication styles in relation to dolphins.
Question 32

Dolphin Communication

The study of cetacean intelligence has added to our understanding of dolphin behavior. Even though cetaceans—marine animals including dolphins, whales, and porpoises—are widely considered intelligent species, conclusions about the type and extent of dolphin intelligence have not yet been reached. However, there are several things that we do know about the ways in which dolphins communicate and behave in groups.

Researchers study the level of communication among dolphins because communication systems can offer insight into an animal’s intelligence. Dolphins produce two primary types of vocalizations called clicks and whistles. Dolphins generally use clicks for the purposes of echolocation and whistles for communication. Dolphins emit clicks as rapid broadband bursts that are sent out into their environment and then echo back, giving them information about their surroundings. Whistles differ from clicks by being sent out as narrow-band frequency modulated signals, which are for communications such as contact calls. Strong evidence supports the idea that dolphins use signature whistles to identify and call to each other.

One hypothesis called the “acoustic flashlight” hypothesis maintains that dolphins may be able to learn passively about their surroundings by listening in on the echolocative inspections of other dolphins. Scientists are researching this idea by looking at how dolphins participate in postural pointing, an activity that humans perform when they point a finger to direct another person towards an object or location.

Communication is also an important part of how dolphins behave in groups, which can range in size from a pair of dolphins to a pod of hundreds of individuals. Researchers have noticed that large packs rely on a highly organized method of communication when responding to predators, such as sharks. When a group of dolphins reacts to an unexpected disturbance, it will quickly move in near-unison to avoid the threat. In this way, dolphins rely on visual and auditory cues to communicate their location and to interpret the locations of others in the pod.
 

The author of this passage is primarily concerned with

A
contrasting dolphin communications with those of other cetaceans.
B
providing an in-depth analysis of a disputed claim.
C
offering an example of how dolphins used highly-developed communication patterns to escape predators.
D
explaining how dolphins communicate and behave in groups.
Question 32 Explanation: 
The correct answer is (D). Notice that each of the passage's paragraphs discuss some aspect of dolphin communication and behavior. The final paragraph explains how dolphins communicate in groups. The final sentence of the first paragraph establishes this focus, saying: "...there are several things that we do know about the ways in which dolphins communicate and behave in groups."
Question 33

Dolphin Communication

The study of cetacean intelligence has added to our understanding of dolphin behavior. Even though cetaceans—marine animals including dolphins, whales, and porpoises—are widely considered intelligent species, conclusions about the type and extent of dolphin intelligence have not yet been reached. However, there are several things that we do know about the ways in which dolphins communicate and behave in groups.

Researchers study the level of communication among dolphins because communication systems can offer insight into an animal’s intelligence. Dolphins produce two primary types of vocalizations called clicks and whistles. Dolphins generally use clicks for the purposes of echolocation and whistles for communication. Dolphins emit clicks as rapid broadband bursts that are sent out into their environment and then echo back, giving them information about their surroundings. Whistles differ from clicks by being sent out as narrow-band frequency modulated signals, which are for communications such as contact calls. Strong evidence supports the idea that dolphins use signature whistles to identify and call to each other.

One hypothesis called the “acoustic flashlight” hypothesis maintains that dolphins may be able to learn passively about their surroundings by listening in on the echolocative inspections of other dolphins. Scientists are researching this idea by looking at how dolphins participate in postural pointing, an activity that humans perform when they point a finger to direct another person towards an object or location.

Communication is also an important part of how dolphins behave in groups, which can range in size from a pair of dolphins to a pod of hundreds of individuals. Researchers have noticed that large packs rely on a highly organized method of communication when responding to predators, such as sharks. When a group of dolphins reacts to an unexpected disturbance, it will quickly move in near-unison to avoid the threat. In this way, dolphins rely on visual and auditory cues to communicate their location and to interpret the locations of others in the pod.
 

The phrase “acoustic flashlight” is an example of

A
a simile
B
a metaphor
C
personification
D
an allusion
Question 33 Explanation: 
The correct answer is (B). The phrase “acoustic flashlight” is a metaphor comparing dolphins pointing their bodies towards what they want to see and people pointing flashlights towards what they want to see. It is called an “acoustic flashlight” because once the dolphin is pointed in the right direction, it sees using sound rather than light.
Question 34

The Most Dangerous Game

“Off there to the right—somewhere—is a large island,” said Whitney. “It's rather a mystery—”

“What island is it?” Rainsford asked.

“The old charts call it ‘Ship-Trap Island’,” Whitney replied. “A suggestive name, isn't it? Sailors have a curious dread of the place. I don't know why. Some superstition—”

“Can't see it,” remarked Rainsford, trying to peer through the dank tropical night that was palpable as it pressed its thick warm blackness in upon the yacht.

“You've good eyes,” said Whitney, with a laugh, “and I've seen you pick off a moose moving in the brown fall bush at four hundred yards, but even you can't see four miles or so through a moonless Caribbean night.”

“Nor four yards,” admitted Rainsford. “Ugh! It's like moist black velvet.”

“It will be light enough in Rio,” promised Whitney. “We should make it in a few days. I hope the jaguar guns have come from Purdey's. We should have some good hunting up the Amazon. Great sport, hunting.”

“The best sport in the world,” agreed Rainsford.

“For the hunter,” amended Whitney. “Not for the jaguar.”

“Don't talk rot, Whitney," said Rainsford. “You're a big-game hunter, not a philosopher. Who cares how a jaguar feels?”

“Perhaps the jaguar does,” observed Whitney.

“Bah! They've no understanding.”

“Even so, I rather think they understand one thing—fear. The fear of pain and the fear of death.”

“Nonsense,” laughed Rainsford. “This hot weather is making you soft, Whitney. Be a realist. The world is made up of two classes—the hunters and the huntees. Luckily, you and I are hunters.”
 

What is the setting for the story?

A
A chilly night in the Amazon rainforest
B
A sweltering afternoon on the Amazon River
C
A humid night on the Caribbean Sea
D
A damp night in Rio de Janeiro
Question 34 Explanation: 
The correct choice is (C). The setting is very clearly established when Whitney tells Rainsford, “...even you can’t see four miles or so through a moonless Caribbean night.”
Question 35

The Most Dangerous Game

“Off there to the right—somewhere—is a large island,” said Whitney. “It's rather a mystery—”

“What island is it?” Rainsford asked.

“The old charts call it ‘Ship-Trap Island’,” Whitney replied. “A suggestive name, isn't it? Sailors have a curious dread of the place. I don't know why. Some superstition—”

“Can't see it,” remarked Rainsford, trying to peer through the dank tropical night that was palpable as it pressed its thick warm blackness in upon the yacht.

“You've good eyes,” said Whitney, with a laugh, “and I've seen you pick off a moose moving in the brown fall bush at four hundred yards, but even you can't see four miles or so through a moonless Caribbean night.”

“Nor four yards,” admitted Rainsford. “Ugh! It's like moist black velvet.”

“It will be light enough in Rio,” promised Whitney. “We should make it in a few days. I hope the jaguar guns have come from Purdey's. We should have some good hunting up the Amazon. Great sport, hunting.”

“The best sport in the world,” agreed Rainsford.

“For the hunter,” amended Whitney. “Not for the jaguar.”

“Don't talk rot, Whitney," said Rainsford. “You're a big-game hunter, not a philosopher. Who cares how a jaguar feels?”

“Perhaps the jaguar does,” observed Whitney.

“Bah! They've no understanding.”

“Even so, I rather think they understand one thing—fear. The fear of pain and the fear of death.”

“Nonsense,” laughed Rainsford. “This hot weather is making you soft, Whitney. Be a realist. The world is made up of two classes—the hunters and the huntees. Luckily, you and I are hunters.”
 

Read this sentence from the passage:

“You've good eyes,” said Whitney, with a laugh, “and I've seen you pick off a moose moving in the brown fall bush at four hundred yards, but even you can't see four miles or so through a moonless Caribbean night.”
 

The author uses this underlined statement to show that:

A
Rainsford is a skilled hunter
B
Rainsford is a lucky shot
C
Whitney idolizes Rainsford
D
Rainsford is a ruthless hunter
Question 35 Explanation: 
The correct answer is (A). The author uses this statement to show that Rainsford is an expert marksman and that he and Whitney have hunted together in the past.
Question 36

The Most Dangerous Game

“Off there to the right—somewhere—is a large island,” said Whitney. “It's rather a mystery—”

“What island is it?” Rainsford asked.

“The old charts call it ‘Ship-Trap Island’,” Whitney replied. “A suggestive name, isn't it? Sailors have a curious dread of the place. I don't know why. Some superstition—”

“Can't see it,” remarked Rainsford, trying to peer through the dank tropical night that was palpable as it pressed its thick warm blackness in upon the yacht.

“You've good eyes,” said Whitney, with a laugh, “and I've seen you pick off a moose moving in the brown fall bush at four hundred yards, but even you can't see four miles or so through a moonless Caribbean night.”

“Nor four yards,” admitted Rainsford. “Ugh! It's like moist black velvet.”

“It will be light enough in Rio,” promised Whitney. “We should make it in a few days. I hope the jaguar guns have come from Purdey's. We should have some good hunting up the Amazon. Great sport, hunting.”

“The best sport in the world,” agreed Rainsford.

“For the hunter,” amended Whitney. “Not for the jaguar.”

“Don't talk rot, Whitney," said Rainsford. “You're a big-game hunter, not a philosopher. Who cares how a jaguar feels?”

“Perhaps the jaguar does,” observed Whitney.

“Bah! They've no understanding.”

“Even so, I rather think they understand one thing—fear. The fear of pain and the fear of death.”

“Nonsense,” laughed Rainsford. “This hot weather is making you soft, Whitney. Be a realist. The world is made up of two classes—the hunters and the huntees. Luckily, you and I are hunters.”
 

How do Rainsford and Whitney's opinions on the jaguar differ?

A
Rainsford doesn’t believe jaguars have feelings.
B
Whitney believes they should consider the feelings of the jaguar.
C
Rainsford believes the jaguars are intelligent.
D
Whitney believes the jaguars lack understanding.
Question 36 Explanation: 
The correct answer is (B). Rainsford and Whitney both indicate they believe the jaguar has emotions, but Rainsford doesn’t believe they have any “understanding.” Rainsford asks “who cares” how the jaguars feel, while Whitney says he believes they understand fear and that perhaps this should be taken into account.
Question 37

The Most Dangerous Game

“Off there to the right—somewhere—is a large island,” said Whitney. “It's rather a mystery—”

“What island is it?” Rainsford asked.

“The old charts call it ‘Ship-Trap Island’,” Whitney replied. “A suggestive name, isn't it? Sailors have a curious dread of the place. I don't know why. Some superstition—”

“Can't see it,” remarked Rainsford, trying to peer through the dank tropical night that was palpable as it pressed its thick warm blackness in upon the yacht.

“You've good eyes,” said Whitney, with a laugh, “and I've seen you pick off a moose moving in the brown fall bush at four hundred yards, but even you can't see four miles or so through a moonless Caribbean night.”

“Nor four yards,” admitted Rainsford. “Ugh! It's like moist black velvet.”

“It will be light enough in Rio,” promised Whitney. “We should make it in a few days. I hope the jaguar guns have come from Purdey's. We should have some good hunting up the Amazon. Great sport, hunting.”

“The best sport in the world,” agreed Rainsford.

“For the hunter,” amended Whitney. “Not for the jaguar.”

“Don't talk rot, Whitney," said Rainsford. “You're a big-game hunter, not a philosopher. Who cares how a jaguar feels?”

“Perhaps the jaguar does,” observed Whitney.

“Bah! They've no understanding.”

“Even so, I rather think they understand one thing—fear. The fear of pain and the fear of death.”

“Nonsense,” laughed Rainsford. “This hot weather is making you soft, Whitney. Be a realist. The world is made up of two classes—the hunters and the huntees. Luckily, you and I are hunters.”
 

The description of the island creates a sense of

A
astonishment
B
bewilderment
C
foreboding
D
irony
Question 37 Explanation: 
The correct answer is (C). Foreboding is a fearful apprehension or a feeling that something bad will happen. With the name Ship-Trap Island and the comment about “a curious dread of the place,” there is a sense that something bad may happen near this island.
Question 38

The Most Dangerous Game

“Off there to the right—somewhere—is a large island,” said Whitney. “It's rather a mystery—”

“What island is it?” Rainsford asked.

“The old charts call it ‘Ship-Trap Island’,” Whitney replied. “A suggestive name, isn't it? Sailors have a curious dread of the place. I don't know why. Some superstition—”

“Can't see it,” remarked Rainsford, trying to peer through the dank tropical night that was palpable as it pressed its thick warm blackness in upon the yacht.

“You've good eyes,” said Whitney, with a laugh, “and I've seen you pick off a moose moving in the brown fall bush at four hundred yards, but even you can't see four miles or so through a moonless Caribbean night.”

“Nor four yards,” admitted Rainsford. “Ugh! It's like moist black velvet.”

“It will be light enough in Rio,” promised Whitney. “We should make it in a few days. I hope the jaguar guns have come from Purdey's. We should have some good hunting up the Amazon. Great sport, hunting.”

“The best sport in the world,” agreed Rainsford.

“For the hunter,” amended Whitney. “Not for the jaguar.”

“Don't talk rot, Whitney," said Rainsford. “You're a big-game hunter, not a philosopher. Who cares how a jaguar feels?”

“Perhaps the jaguar does,” observed Whitney.

“Bah! They've no understanding.”

“Even so, I rather think they understand one thing—fear. The fear of pain and the fear of death.”

“Nonsense,” laughed Rainsford. “This hot weather is making you soft, Whitney. Be a realist. The world is made up of two classes—the hunters and the huntees. Luckily, you and I are hunters.”
 

As it is used in the last paragraph, the word “soft” most nearly means

A
squashy
B
dim
C
subtle
D
sympathetic
Question 38 Explanation: 
The correct answer is (D). When Rainsford calls Whitney “soft,” he means that Whitney has let his emotions make him overly sympathetic to the jaguar.
Question 39

The Most Dangerous Game

“Off there to the right—somewhere—is a large island,” said Whitney. “It's rather a mystery—”

“What island is it?” Rainsford asked.

“The old charts call it ‘Ship-Trap Island’,” Whitney replied. “A suggestive name, isn't it? Sailors have a curious dread of the place. I don't know why. Some superstition—”

“Can't see it,” remarked Rainsford, trying to peer through the dank tropical night that was palpable as it pressed its thick warm blackness in upon the yacht.

“You've good eyes,” said Whitney, with a laugh, “and I've seen you pick off a moose moving in the brown fall bush at four hundred yards, but even you can't see four miles or so through a moonless Caribbean night.”

“Nor four yards,” admitted Rainsford. “Ugh! It's like moist black velvet.”

“It will be light enough in Rio,” promised Whitney. “We should make it in a few days. I hope the jaguar guns have come from Purdey's. We should have some good hunting up the Amazon. Great sport, hunting.”

“The best sport in the world,” agreed Rainsford.

“For the hunter,” amended Whitney. “Not for the jaguar.”

“Don't talk rot, Whitney," said Rainsford. “You're a big-game hunter, not a philosopher. Who cares how a jaguar feels?”

“Perhaps the jaguar does,” observed Whitney.

“Bah! They've no understanding.”

“Even so, I rather think they understand one thing—fear. The fear of pain and the fear of death.”

“Nonsense,” laughed Rainsford. “This hot weather is making you soft, Whitney. Be a realist. The world is made up of two classes—the hunters and the huntees. Luckily, you and I are hunters.”
 

Which of the following best describes Rainsford's character?

A
realistic and blunt
B
bullying and aggressive
C
sarcastic and jaded
D
philosophical and practical
Question 39 Explanation: 
The correct answer is (A). Rainsford tells Whitney to be “a realist.” He obviously thinks this is the best way to be, and it is most likely how he himself views the world. He is very direct, or blunt, in how he speaks to Whitney, correcting him and offering his own opinions without bullying him. Whitney is the one who is more philosophical and practical.
Question 40

Recycling Programs

Whether it’s saving milk jugs, sorting newspapers neatly into a pile, or placing unnecessary office paper in a corner recycling bin, the American recycling experiment continues. Consider the set of recycling statistics, reflected in the bar chart. It compares American recycling rates for select materials (paper, glass, metals and plastics) over a fifty year time frame (1960–2010), using ten year intervals.



The large green bars on the graph show that between 1960 and 2010, paper recycling rates exceeded the recycling rates for the other materials. As the years pass, American recycling habits expanded, with beverage container recycling explaining much of the increase in glass, metals and plastics recycling in 1990. Starting in 1990, yard trimming recycling rates, not presented in the top bar chart, also occupied a larger portion of the average American’s recycling efforts. By 2010, Americans were recycling 57.5% of all their yard trimmings.

In many locations, changing technology and community practices contributed to recycling rate upward momentum over this same sixty year time frame. Reverse vending machines, invented during a 1990s recycling technology wave, now fill space in many retail locations around the country. State beverage container recycling laws and ease of use account for a portion of their long term success.

While circumstances exist where individuals might need a moment to stop and think through any particular recycling task, most modern recycling tasks, like using reverse vending machines, are quite simple, and accomplished by many individuals unreflective participation in organized beverage container recycling programs.

The 9,000 curbside recycling programs in existence between 1985 and 2005 also contributed to increased aggregate recycling rates. Curbside recycling schedules that run concurrent with local garbage collection schedules allow households to schedule garbage and recycling chores for the same day. All curbside recycling programs follow some general rules. Five of the most common are presented below.

  • Follow Sorting Guidelines
  • Keep Recycled Material Clean
  • Know Your Recycling Bins
  • Know Your Recycling Day
  • In Doubt, Leave it Out
Creating a successful home, school, or work recycling program takes very little effort, while creating substantial environmental and economic benefits. Recycling practices easily blend into modern American life. Most successful recycling programs begin and end with locating their recycling corner. Strategically placing a recycling center in a corner of a high traffic location often works to attract individual attention along with providing a centralized waste removal location.
 

According to the chart provided, which of the following materials was most commonly recycled in 2010?

A
Paper/Paperboard
B
Metals
C
Glass
D
Plastics
Question 40 Explanation: 
The correct answer is (A). In 2010, the green bar, representing Paper/Paperboard recycling, is higher than the other bars in the year 2010.
Question 41

Recycling Programs

Whether it’s saving milk jugs, sorting newspapers neatly into a pile, or placing unnecessary office paper in a corner recycling bin, the American recycling experiment continues. Consider the set of recycling statistics, reflected in the bar chart. It compares American recycling rates for select materials (paper, glass, metals and plastics) over a fifty year time frame (1960–2010), using ten year intervals.



The large green bars on the graph show that between 1960 and 2010, paper recycling rates exceeded the recycling rates for the other materials. As the years pass, American recycling habits expanded, with beverage container recycling explaining much of the increase in glass, metals and plastics recycling in 1990. Starting in 1990, yard trimming recycling rates, not presented in the top bar chart, also occupied a larger portion of the average American’s recycling efforts. By 2010, Americans were recycling 57.5% of all their yard trimmings.

In many locations, changing technology and community practices contributed to recycling rate upward momentum over this same sixty year time frame. Reverse vending machines, invented during a 1990s recycling technology wave, now fill space in many retail locations around the country. State beverage container recycling laws and ease of use account for a portion of their long term success.

While circumstances exist where individuals might need a moment to stop and think through any particular recycling task, most modern recycling tasks, like using reverse vending machines, are quite simple, and accomplished by many individuals unreflective participation in organized beverage container recycling programs.

The 9,000 curbside recycling programs in existence between 1985 and 2005 also contributed to increased aggregate recycling rates. Curbside recycling schedules that run concurrent with local garbage collection schedules allow households to schedule garbage and recycling chores for the same day. All curbside recycling programs follow some general rules. Five of the most common are presented below.

  • Follow Sorting Guidelines
  • Keep Recycled Material Clean
  • Know Your Recycling Bins
  • Know Your Recycling Day
  • In Doubt, Leave it Out
Creating a successful home, school, or work recycling program takes very little effort, while creating substantial environmental and economic benefits. Recycling practices easily blend into modern American life. Most successful recycling programs begin and end with locating their recycling corner. Strategically placing a recycling center in a corner of a high traffic location often works to attract individual attention along with providing a centralized waste removal location.
 

According to the text, what are TWO factors that have made reverse vending machines a success?

A
state laws
B
customer payments
C
federal regulations
D
ease of use
E
habits
F
curbside recycling programs
Question 41 Explanation: 
The correct answers are (A) and (D). The phrase “according to the text” indicates that the correct answer is explicitly stated in the passage. Skim through to find where the passage mentions reverse vending machines and read carefully to find the answer. After mentioning reverse vending machines, the article reads, “State beverage container recycling laws and ease of use account for a portion of their long term success.”
Question 42

Recycling Programs

Whether it’s saving milk jugs, sorting newspapers neatly into a pile, or placing unnecessary office paper in a corner recycling bin, the American recycling experiment continues. Consider the set of recycling statistics, reflected in the bar chart. It compares American recycling rates for select materials (paper, glass, metals and plastics) over a fifty year time frame (1960–2010), using ten year intervals.



The large green bars on the graph show that between 1960 and 2010, paper recycling rates exceeded the recycling rates for the other materials. As the years pass, American recycling habits expanded, with beverage container recycling explaining much of the increase in glass, metals and plastics recycling in 1990. Starting in 1990, yard trimming recycling rates, not presented in the top bar chart, also occupied a larger portion of the average American’s recycling efforts. By 2010, Americans were recycling 57.5% of all their yard trimmings.

In many locations, changing technology and community practices contributed to recycling rate upward momentum over this same sixty year time frame. Reverse vending machines, invented during a 1990s recycling technology wave, now fill space in many retail locations around the country. State beverage container recycling laws and ease of use account for a portion of their long term success.

While circumstances exist where individuals might need a moment to stop and think through any particular recycling task, most modern recycling tasks, like using reverse vending machines, are quite simple, and accomplished by many individuals unreflective participation in organized beverage container recycling programs.

The 9,000 curbside recycling programs in existence between 1985 and 2005 also contributed to increased aggregate recycling rates. Curbside recycling schedules that run concurrent with local garbage collection schedules allow households to schedule garbage and recycling chores for the same day. All curbside recycling programs follow some general rules. Five of the most common are presented below.

  • Follow Sorting Guidelines
  • Keep Recycled Material Clean
  • Know Your Recycling Bins
  • Know Your Recycling Day
  • In Doubt, Leave it Out
Creating a successful home, school, or work recycling program takes very little effort, while creating substantial environmental and economic benefits. Recycling practices easily blend into modern American life. Most successful recycling programs begin and end with locating their recycling corner. Strategically placing a recycling center in a corner of a high traffic location often works to attract individual attention along with providing a centralized waste removal location.
 

Read this excerpt from the text

The 9,000 curbside recycling programs in existence between 1985 and 2005 also contributed to increased aggregate recycling rates.
 

As used in the excerpt, what is the meaning of the word aggregate?

A
community
B
plastic
C
total
D
official
Question 42 Explanation: 
The correct answer is (C). As used in this context, aggregate means: the total sum after adding together two or more amounts. The article is referring to the overall increase in recycling rates of all items combined.
Question 43

Recycling Programs

Whether it’s saving milk jugs, sorting newspapers neatly into a pile, or placing unnecessary office paper in a corner recycling bin, the American recycling experiment continues. Consider the set of recycling statistics, reflected in the bar chart. It compares American recycling rates for select materials (paper, glass, metals and plastics) over a fifty year time frame (1960–2010), using ten year intervals.



The large green bars on the graph show that between 1960 and 2010, paper recycling rates exceeded the recycling rates for the other materials. As the years pass, American recycling habits expanded, with beverage container recycling explaining much of the increase in glass, metals and plastics recycling in 1990. Starting in 1990, yard trimming recycling rates, not presented in the top bar chart, also occupied a larger portion of the average American’s recycling efforts. By 2010, Americans were recycling 57.5% of all their yard trimmings.

In many locations, changing technology and community practices contributed to recycling rate upward momentum over this same sixty year time frame. Reverse vending machines, invented during a 1990s recycling technology wave, now fill space in many retail locations around the country. State beverage container recycling laws and ease of use account for a portion of their long term success.

While circumstances exist where individuals might need a moment to stop and think through any particular recycling task, most modern recycling tasks, like using reverse vending machines, are quite simple, and accomplished by many individuals unreflective participation in organized beverage container recycling programs.

The 9,000 curbside recycling programs in existence between 1985 and 2005 also contributed to increased aggregate recycling rates. Curbside recycling schedules that run concurrent with local garbage collection schedules allow households to schedule garbage and recycling chores for the same day. All curbside recycling programs follow some general rules. Five of the most common are presented below.

  • Follow Sorting Guidelines
  • Keep Recycled Material Clean
  • Know Your Recycling Bins
  • Know Your Recycling Day
  • In Doubt, Leave it Out
Creating a successful home, school, or work recycling program takes very little effort, while creating substantial environmental and economic benefits. Recycling practices easily blend into modern American life. Most successful recycling programs begin and end with locating their recycling corner. Strategically placing a recycling center in a corner of a high traffic location often works to attract individual attention along with providing a centralized waste removal location.
 

Read this excerpt from the text

Curbside recycling schedules that run concurrent with local garbage collection schedules allow households to schedule garbage and recycling chores for the same day.
 

How does the author use this statement to develop the argument that recycling is a relatively easy activity for the average person?

A
By providing the specific collection times for the recycling bins, and indicating what is picked up on which days.
B
By demonstrating how materials are cleaned, sorted, and placed into specific bins.
C
By outlining the various bins required for recycling and noting how simple it is to sort the recycled materials.
D
By showing there is minimal inconvenience to homeowners to recycle, since they already put out garbage bins once a week.
Question 43 Explanation: 
The correct answer is (D). No specific information about sorting is provided, and the other incorrect answer choices describe processes that are relatively complicated when compared with putting out garbage bins once a week. The alignment of the curbside recycling schedules with the local garbage collection schedules makes it easy for households to simultaneously dispose of trash while they recycle.
Question 44

Recycling Programs

Whether it’s saving milk jugs, sorting newspapers neatly into a pile, or placing unnecessary office paper in a corner recycling bin, the American recycling experiment continues. Consider the set of recycling statistics, reflected in the bar chart. It compares American recycling rates for select materials (paper, glass, metals and plastics) over a fifty year time frame (1960–2010), using ten year intervals.



The large green bars on the graph show that between 1960 and 2010, paper recycling rates exceeded the recycling rates for the other materials. As the years pass, American recycling habits expanded, with beverage container recycling explaining much of the increase in glass, metals and plastics recycling in 1990. Starting in 1990, yard trimming recycling rates, not presented in the top bar chart, also occupied a larger portion of the average American’s recycling efforts. By 2010, Americans were recycling 57.5% of all their yard trimmings.

In many locations, changing technology and community practices contributed to recycling rate upward momentum over this same sixty year time frame. Reverse vending machines, invented during a 1990s recycling technology wave, now fill space in many retail locations around the country. State beverage container recycling laws and ease of use account for a portion of their long term success.

While circumstances exist where individuals might need a moment to stop and think through any particular recycling task, most modern recycling tasks, like using reverse vending machines, are quite simple, and accomplished by many individuals unreflective participation in organized beverage container recycling programs.

The 9,000 curbside recycling programs in existence between 1985 and 2005 also contributed to increased aggregate recycling rates. Curbside recycling schedules that run concurrent with local garbage collection schedules allow households to schedule garbage and recycling chores for the same day. All curbside recycling programs follow some general rules. Five of the most common are presented below.

  • Follow Sorting Guidelines
  • Keep Recycled Material Clean
  • Know Your Recycling Bins
  • Know Your Recycling Day
  • In Doubt, Leave it Out
Creating a successful home, school, or work recycling program takes very little effort, while creating substantial environmental and economic benefits. Recycling practices easily blend into modern American life. Most successful recycling programs begin and end with locating their recycling corner. Strategically placing a recycling center in a corner of a high traffic location often works to attract individual attention along with providing a centralized waste removal location.
 

In the first paragraph, the author uses the phrase American recycling experiment to show

A
that Americans will only recycle on an experimental basis
B
that America has not figured out exactly how recycling programs should be implemented
C
that recycling in America will only continue until the experiment is completed
D
that other countries are more committed to recycling than America
Question 44 Explanation: 
The correct answer is (B). In the context of this passage, the word experiment means "the process of testing." The author uses this phrase to suggest that there is an ongoing effort in America to figure out the best policies for recycling programs and the best ways to encourage household participation.
Question 45

The History of Girl Scout Cookies

For nearly 100 years, the Girl Scouts and their supporters have made their annual cookie sale into an iconic American tradition—and all while they learned valuable life lessons, made their communities better, and most of all: had fun.

Girl Scout Cookies began long ago in the kitchens of troop members, with moms volunteering to help advise. In 1917, only 5 years after the Girl Scouts of America was founded by Juliette Gordon Low, the Mistletoe Troop in Muskogee, Oklahoma began baking and selling cookies in their high school cafeteria as a service project. From these humble beginnings, a national fundraising phenomenon was born.

By 1922, the Girls Scouts of America were getting the word out about this amazing fundraiser. The American Girl magazine (published by GSA), featured an article by Florence E. Neill, a local director in Chicago, Illinois, which provided a cookie recipe complete with a sales plan. In 1933, the Greater Philadelphia Council began baking cookies and selling them in the city’s gas and electric company windows in 1933. Local Girl Scout troops raised money and developed marketing and business skills. By 1934, they became the first council to sell commercially baked cookies.

The Girl Scout Federation of Greater New York soon followed Philadelphia’s lead in 1935, but they added the words “Girl Scout Cookies” to their box. Within a year, the national Girl Scout organization had realized the potential shown in these cities, and began licensing the production of cookies to be sold nationwide. The national excitement for Girl Scout cookies built from there, and by 1951 Girl Scout cookies came in three varieties: Sandwich, Shortbread, and Chocolate Mints. By the 1970’s, the GSA was selling 8 different varieties of cookies.

In more recent years, the GSA has begun to focus more heavily on design, creating bold and bright boxes that captured the spirit of Girl Scouting. There are still 8 varieties of cookies, but now they’re kosher and, much to the excitement of our youngest Girl Scouts, Daisies started selling cookies!
 

Read this excerpt from the text:

For nearly 100 years, the Girl Scouts and their supporters have made their annual cookie sale into an iconic American tradition—and all while they learned valuable life lessons, made their communities better, and most of all: had fun.
 

The most likely meaning of iconic is?

A
required by a rule
B
pertaining to charity
C
happening once a year
D
widely recognized and well-established
Question 45 Explanation: 
The correct answer is (D). An icon is something that is revered or idolized. Iconic means "widely known and acknowledged, especially for distinctive excellence." There is no mention of a rule requiring the cookie sale to take place, nor mention of the sale functioning as a charity. The word annual, not iconic, indicates that the sale happens once a year.
Question 46

The History of Girl Scout Cookies

For nearly 100 years, the Girl Scouts and their supporters have made their annual cookie sale into an iconic American tradition—and all while they learned valuable life lessons, made their communities better, and most of all: had fun.

Girl Scout Cookies began long ago in the kitchens of troop members, with moms volunteering to help advise. In 1917, only 5 years after the Girl Scouts of America was founded by Juliette Gordon Low, the Mistletoe Troop in Muskogee, Oklahoma began baking and selling cookies in their high school cafeteria as a service project. From these humble beginnings, a national fundraising phenomenon was born.

By 1922, the Girls Scouts of America were getting the word out about this amazing fundraiser. The American Girl magazine (published by GSA), featured an article by Florence E. Neill, a local director in Chicago, Illinois, which provided a cookie recipe complete with a sales plan. In 1933, the Greater Philadelphia Council began baking cookies and selling them in the city’s gas and electric company windows in 1933. Local Girl Scout troops raised money and developed marketing and business skills. By 1934, they became the first council to sell commercially baked cookies.

The Girl Scout Federation of Greater New York soon followed Philadelphia’s lead in 1935, but they added the words “Girl Scout Cookies” to their box. Within a year, the national Girl Scout organization had realized the potential shown in these cities, and began licensing the production of cookies to be sold nationwide. The national excitement for Girl Scout cookies built from there, and by 1951 Girl Scout cookies came in three varieties: Sandwich, Shortbread, and Chocolate Mints. By the 1970’s, the GSA was selling 8 different varieties of cookies.

In more recent years, the GSA has begun to focus more heavily on design, creating bold and bright boxes that captured the spirit of Girl Scouting. There are still 8 varieties of cookies, but now they’re kosher and, much to the excitement of our youngest Girl Scouts, Daisies started selling cookies!
 

Which detail in the passage supports the idea that the tradition of selling Girl Scout cookies started as a local project before it caught on within the GSA nationally?

A
“…from these humble beginnings, a national fundraising phenomenon was born” (paragraph 2).
B
“In more recent years, the GSA has begun to focus more heavily on design…” (paragraph 5).
C
“by 1951 Girl Scout cookies came in three varieties: Sandwich, Shortbread, and Chocolate Mints” (paragraph 4).
D
“For nearly 100 years, the Girl Scouts and their supporters have made their annual cookie sale into an iconic American tradition” (paragraph 1).
Question 46 Explanation: 
The correct answer is (A) because it is the only detail that addresses the fact that the national tradition of selling Girl Scout cookies began with a local troop. None of the other answer choices address the tradition’s local roots.
Question 47

The History of Girl Scout Cookies

For nearly 100 years, the Girl Scouts and their supporters have made their annual cookie sale into an iconic American tradition—and all while they learned valuable life lessons, made their communities better, and most of all: had fun.

Girl Scout Cookies began long ago in the kitchens of troop members, with moms volunteering to help advise. In 1917, only 5 years after the Girl Scouts of America was founded by Juliette Gordon Low, the Mistletoe Troop in Muskogee, Oklahoma began baking and selling cookies in their high school cafeteria as a service project. From these humble beginnings, a national fundraising phenomenon was born.

By 1922, the Girls Scouts of America were getting the word out about this amazing fundraiser. The American Girl magazine (published by GSA), featured an article by Florence E. Neill, a local director in Chicago, Illinois, which provided a cookie recipe complete with a sales plan. In 1933, the Greater Philadelphia Council began baking cookies and selling them in the city’s gas and electric company windows in 1933. Local Girl Scout troops raised money and developed marketing and business skills. By 1934, they became the first council to sell commercially baked cookies.

The Girl Scout Federation of Greater New York soon followed Philadelphia’s lead in 1935, but they added the words “Girl Scout Cookies” to their box. Within a year, the national Girl Scout organization had realized the potential shown in these cities, and began licensing the production of cookies to be sold nationwide. The national excitement for Girl Scout cookies built from there, and by 1951 Girl Scout cookies came in three varieties: Sandwich, Shortbread, and Chocolate Mints. By the 1970’s, the GSA was selling 8 different varieties of cookies.

In more recent years, the GSA has begun to focus more heavily on design, creating bold and bright boxes that captured the spirit of Girl Scouting. There are still 8 varieties of cookies, but now they’re kosher and, much to the excitement of our youngest Girl Scouts, Daisies started selling cookies!
 

Which idea about the history of the Girl Scout cookie is supported by the passage?

A
A troop in Oklahoma was the first to use commercial bakers in their fundraising efforts.
B
The GSA is known for putting little effort into the design and presentation of their product.
C
A troop in Philadelphia was the first to use commercial bakers in their fundraising efforts.
D
The GSA have always been focused on how the design of their cookie boxes impacts their cookie sales.
Question 47 Explanation: 
The correct answer is (C). While the passage states that the tradition of baking and selling cookies began in Oklahoma, it specifically asserts that a troop in greater Philadelphia was the first to use commercial bakers. Answer choices (B) and (D) are both incorrect because the passage implies that the GSA only began focusing on the design of their boxes in the early 2000’s, which means that they have not “always been focused” (D) on it, nor have they put “little effort” (B) into their box design.
Question 48

The History of Girl Scout Cookies

For nearly 100 years, the Girl Scouts and their supporters have made their annual cookie sale into an iconic American tradition—and all while they learned valuable life lessons, made their communities better, and most of all: had fun.

Girl Scout Cookies began long ago in the kitchens of troop members, with moms volunteering to help advise. In 1917, only 5 years after the Girl Scouts of America was founded by Juliette Gordon Low, the Mistletoe Troop in Muskogee, Oklahoma began baking and selling cookies in their high school cafeteria as a service project. From these humble beginnings, a national fundraising phenomenon was born.

By 1922, the Girls Scouts of America were getting the word out about this amazing fundraiser. The American Girl magazine (published by GSA), featured an article by Florence E. Neill, a local director in Chicago, Illinois, which provided a cookie recipe complete with a sales plan. In 1933, the Greater Philadelphia Council began baking cookies and selling them in the city’s gas and electric company windows in 1933. Local Girl Scout troops raised money and developed marketing and business skills. By 1934, they became the first council to sell commercially baked cookies.

The Girl Scout Federation of Greater New York soon followed Philadelphia’s lead in 1935, but they added the words “Girl Scout Cookies” to their box. Within a year, the national Girl Scout organization had realized the potential shown in these cities, and began licensing the production of cookies to be sold nationwide. The national excitement for Girl Scout cookies built from there, and by 1951 Girl Scout cookies came in three varieties: Sandwich, Shortbread, and Chocolate Mints. By the 1970’s, the GSA was selling 8 different varieties of cookies.

In more recent years, the GSA has begun to focus more heavily on design, creating bold and bright boxes that captured the spirit of Girl Scouting. There are still 8 varieties of cookies, but now they’re kosher and, much to the excitement of our youngest Girl Scouts, Daisies started selling cookies!
 

Read this sentence from Paragraph 4:

The Girl Scout Federation of Greater New York soon followed Philadelphia’s lead in 1935, but they added the words “Girl Scout Cookies” to their box.
 

What can readers infer from this sentence?

A
Girl Scouts in New York are better marketers than Girl Scouts in Philadelphia.
B
Selling cookies didn’t become profitable for Girl Scout troops until 1935.
C
Girl Scouts in Philadelphia are better marketers than Girl Scouts in New York.
D
The name “Girl Scout Cookies,” which is so well-known now, wasn’t created until 1935.
Question 48 Explanation: 
The correct answer is (D). An argument could be made for either choice (A) or (C) to be correct, however readers don’t have enough information to definitively infer which troop is best at marketing their product. Answer choice (B) is not supported by the passage.
Question 49

The History of Girl Scout Cookies

For nearly 100 years, the Girl Scouts and their supporters have made their annual cookie sale into an iconic American tradition—and all while they learned valuable life lessons, made their communities better, and most of all: had fun.

Girl Scout Cookies began long ago in the kitchens of troop members, with moms volunteering to help advise. In 1917, only 5 years after the Girl Scouts of America was founded by Juliette Gordon Low, the Mistletoe Troop in Muskogee, Oklahoma began baking and selling cookies in their high school cafeteria as a service project. From these humble beginnings, a national fundraising phenomenon was born.

By 1922, the Girls Scouts of America were getting the word out about this amazing fundraiser. The American Girl magazine (published by GSA), featured an article by Florence E. Neill, a local director in Chicago, Illinois, which provided a cookie recipe complete with a sales plan. In 1933, the Greater Philadelphia Council began baking cookies and selling them in the city’s gas and electric company windows in 1933. Local Girl Scout troops raised money and developed marketing and business skills. By 1934, they became the first council to sell commercially baked cookies.

The Girl Scout Federation of Greater New York soon followed Philadelphia’s lead in 1935, but they added the words “Girl Scout Cookies” to their box. Within a year, the national Girl Scout organization had realized the potential shown in these cities, and began licensing the production of cookies to be sold nationwide. The national excitement for Girl Scout cookies built from there, and by 1951 Girl Scout cookies came in three varieties: Sandwich, Shortbread, and Chocolate Mints. By the 1970’s, the GSA was selling 8 different varieties of cookies.

In more recent years, the GSA has begun to focus more heavily on design, creating bold and bright boxes that captured the spirit of Girl Scouting. There are still 8 varieties of cookies, but now they’re kosher and, much to the excitement of our youngest Girl Scouts, Daisies started selling cookies!
 

How does the final paragraph of the passage relate to the rest of the passage?

A
It summarizes the history of Girl Scout cookies.
B
It provides a counterargument to solidify the position of the writer.
C
It provides the most recent chronological information in the passage.
D
It calls the audience to action.
Question 49 Explanation: 
The correct answer is (C). The entire passage is set up like a chronological telling of the story of Girl Scout cookies. As such, the final paragraph provides the most recent information. None of the other answer choices reflect the tone and purpose of the passage.
Question 50

The History of Girl Scout Cookies

For nearly 100 years, the Girl Scouts and their supporters have made their annual cookie sale into an iconic American tradition—and all while they learned valuable life lessons, made their communities better, and most of all: had fun.

Girl Scout Cookies began long ago in the kitchens of troop members, with moms volunteering to help advise. In 1917, only 5 years after the Girl Scouts of America was founded by Juliette Gordon Low, the Mistletoe Troop in Muskogee, Oklahoma began baking and selling cookies in their high school cafeteria as a service project. From these humble beginnings, a national fundraising phenomenon was born.

By 1922, the Girls Scouts of America were getting the word out about this amazing fundraiser. The American Girl magazine (published by GSA), featured an article by Florence E. Neill, a local director in Chicago, Illinois, which provided a cookie recipe complete with a sales plan. In 1933, the Greater Philadelphia Council began baking cookies and selling them in the city’s gas and electric company windows in 1933. Local Girl Scout troops raised money and developed marketing and business skills. By 1934, they became the first council to sell commercially baked cookies.

The Girl Scout Federation of Greater New York soon followed Philadelphia’s lead in 1935, but they added the words “Girl Scout Cookies” to their box. Within a year, the national Girl Scout organization had realized the potential shown in these cities, and began licensing the production of cookies to be sold nationwide. The national excitement for Girl Scout cookies built from there, and by 1951 Girl Scout cookies came in three varieties: Sandwich, Shortbread, and Chocolate Mints. By the 1970’s, the GSA was selling 8 different varieties of cookies.

In more recent years, the GSA has begun to focus more heavily on design, creating bold and bright boxes that captured the spirit of Girl Scouting. There are still 8 varieties of cookies, but now they’re kosher and, much to the excitement of our youngest Girl Scouts, Daisies started selling cookies!
 

Read this sentence from paragraph 5:

There are still 8 varieties of cookies, but now they’re kosher and, much to the excitement of our youngest Girl Scouts, Daisies started selling cookies!
 

Replacing the word “excitement” with “astonishment” changes the meaning of the sentence to suggest that allowing Daisies to sell cookies was

A
overdue
B
unexpected
C
unpopular
D
careless
Question 50 Explanation: 
The correct answer is (B). Astonishment is a feeling associated with surprise. Using the word astonishment would suggest that the change was completely unexpected.
Once you are finished, click the button below. Any items you have not completed will be marked incorrect. Get Results
There are 50 questions to complete.
List
Return
Shaded items are complete.
12345
678910
1112131415
1617181920
2122232425
2627282930
3132333435
3637383940
4142434445
4647484950
End
Return